Вы находитесь на странице: 1из 60

1. A 13 year-old boy is having a bloody diarrhea 5 times a day for 2 days. He vomits 3 times.

He felt pain in his anus while he passes his stool. He also has fever. What mechanism
occur in his bowel when passing the stool?
a. Longitudinal muscle relaxation in receiving segment
b. Circular muscle contraction in receiving segment
c. Longitudinal muscle contraction in propulsive segment
d. Circular muscle contraction in propulsive segment
e. Circular muscle relaxation in propulsive segment

2. A 34-year-old woman has the sudden onset of severe abdominal pain. On physical
examination she is afebrile. The pain is centered in the mid-epigastric region, though there
is marked diffuse tenderness in all quadrants. Laboratory studies show her serum amylase
is 410 U/L and lipase is 610 U/L. Which of the following laboratory test findings is most
likely to be present in this woman?
a. Hypoproteinemia
b. Positive urea breath test
c. Hyperglycemia
d. Elevated sweat chloride
e. Positive serology for HBsAg

3. A 31-year-old woman has a 10 year history of intermittent, bloody diarrhea. She has no
other major medical problems. On physical examination there are no lesions palpable on
digital rectal examination, but a stool sample is positive for occult blood
This patient is at greatest risk for development of which of the following conditions?
A Pancreatitis
B Diverticulitis
C Cholangitis
D Appendicitis
E Perirectal fistula
4. A 32 year old woman comes to the emergency room with severe epigastric and right
upper quadrant pain 3 hours after eating gulai in Minang restaurant. The pain occasionally radiates to
her right flank and right back. She has slight fever and her abdomen is tender to palpation in the
right upper quadrant. Her lab tests are as follows:
ALT: 44 U/l (<30 )
AST: 25 U/l (<30 )
Total bilirubin: 1,3 mg/dl (0-1)
Alkaline phosphatase: 100 U/l (38-126)
White blood 14500 (6000-10000)
Amylase: 67 U/l (53-123)
Lipase: 55 U/l (10-150)

What is the most likely explanation for this patient’s symptoms?


a. Acute cholecystitis
b. Acute peptic ulcer
C. Acute pancreatitis
D. Acute appendicitis
E. Acute hepatitis B

5. Adi was a 25-year-old male who complained of pain, nausea with vomiting, and
tenderness in the right lower quadrant. The pain was first vague and diffuse, and then
became more severe in the epigastric region before localizing in the right lower quadrant.
The pain was getting worse with movement, coughing, or sneezing.. His vital sign were: BP
140/90, Pulse 96x/min, RR 18x/min, Temperature 38.2°C. A marked tenderness was found
on the right lower quadrant , particularly when pressed and released. There was a
decreased of bowel sounds.

What is the most suitable next step management for this patient?
a. Abdominal X-ray
b. Complete Blood count
c. Insert Nasogastric tube
d. Establish an intravenousline
e. Prepare immediate operation
6. A 52-year-old man with peptic ulcer disease has been on drug therapy for 3 months. While
on this regimen, he noticed changes in his bowel habits, increasing headaches, dizziness,
skin rashes, loss of libido, and gynecomastia. Which drug is the most likely responsible for
these effects?
a. cimetidine
b. famotidine
c. misoprostol
d.omeprazole
e. ranitidine

7. A 62-year-old man with chief complain nausea,vomiting, and epigastric pain. He was given
stomach medication, three hour later, he had dyskinesia and rigidity (extra pyramidal
symptoms). Which of the following drug is the most appropriate that can cause this
situation?
A. domperidone
B. metoclopramide
C. ondansetrone
D. omeprazole
E. sucralfate

8. A 48-year-old man presents with a complaint of chronic no bloody diarrhea and right
lower quadrant pain with a palpable mass and tenderness. He states that this "flare-up" is
one of the worst he has ever experienced. Radiographic exam reveals evidence of
ulceration, stricturing, and fistula development of the colon and small bowel.
Which of the following drugs would be the most appropriate for treating this patient?
A. Diphenoxylate
B. Loperamide
C. Hyoscyamine
D. Mesalamine(utk ulcerative colitis) gambaran xray haustra hilang
E. Prednisone

9. A 70-year-old man with a history of atrial fibrillation is started on an oral anticoagulant. His
prothrombin time is monitored on a regular basis. A few months into his therapy, he
begins treatment for a duodenal ulcer and he develops symptoms of a bleeding diathesis.
Which of the following ulcer medications is most likely responsible for this change in his
hemostatic status?
A. Cimetidine
B. Famotidine
C. Misoprostol
D. Rabeprazole
E. Ranitidine

10. A 51-year-old man comes to your private practice with a history of mid epigastric upper
abdominal pain that diminished by eating or taking antacid tables. He drinks 2-3 glasses of
beer every night, also taking 2-3 tablets of aspirin frequently for his stress related
headache. PE reveals no evidence for blood in the stool. You advised him to replace the
aspirin with acetaminophen and stop his drinking of beer; you give him a prescription of
famotidine 20 mg bid. Which of the following is the mechanism of action of famotidine?
a. Eradication of H.pylori
b. Increase of motility
c. Inhibit acid secretion
d. Neutralized gastric acid
e. Protect gastric mucous

11. A 48-year-old woman recently diagnosed with an adenocarcinoma. The oncologist


schedules her for radiation and chemotherapy. Because nausea and vomiting are common
complications, you recommend antiemetic premedication with dexamethasone. Which of
the following antiemetic drugs would be the most appropriate for this condition?
a. Chlorpromazine
b. Dimenhydrinate
c. Ondansetron
d. Prometazine
e. Scopolamine

12. A 24 year old woman comes to your private practice because of experiencing heartburn
after meals, especially when reclining on her sectional at home. Her symptoms are
typically relieved by antacids. In addition to an H2 blockers or PPI drugs, what additional
agent could help treat her esophagitis by coating necrotic tissue with a protective barrier,
and thereby promote healing of the ulcer?
a. Attapulgite
b. Calcium carbonate
c. Loperamide
d. Mg-Al hydroxide
e. Sucralfate

13. A 18-year-old man presents with intermittent cramp abdominal pain, no bloody diarrhea,
and weight loss of 5 kg over 6 months. The diarrhea, wake him from sleep. On a few
occasions, he has had fevers, nausea, and vomiting. The patient describes his work as
being stressful. PE: normal vital signs. He has an oral aphthous ulcer (sariawan) and poorly
localized lower abdominal to mid abdominal tenderness without peritoneal signs. Anal
and rectal examinations are normal, and a stool guaiac test (for detecting occult blood) is
negative. Stool leukocytes are present. The Hb 11.5 and Ht is 34%. Results of
sigmoidoscopy are normal. Which of the following is the most likely diagnosis for this
patient?
a. Acute appendicitis
b. Crohn disease
c. Colon cancer
d. Irritable bowel syndrome
e. Ulcerative colitis

14. A 52-year-old man with a history of poorly controlled diabetes mellitus presents to the
emergency department with severe abdominal pain of 36 hours’ duration. He had no
fever, some times drink ½ glass of beer. A similar episode 4 months ago. PE: tachycardia,
diminished bowel sounds, epigastric tenderness, and a papular rash on his knees.
Laboratory studies are significant for the following: leukocytes, 15,000 cells/mm3; blood
glucose level, 450 mg/dl; amylase level slightly increased. Which of the following is the
most likely diagnosis?
a. Acute on chronic pancreatitis
b. Pancreatitis due to gallstone
c. Pancreatitis due to alcohol
d. Pancreatitis due to infection
e. Pancreatitis due to hypertriglyceridemia

15. A 50-year-old man comes to your clinic complaining of acute severe upper abdominal pain
that radiates to his back and worsens with meals. He has a long history of binge drinking.
He notes that lately he has been losing weight and that his stools have been loose.
Physical examination reveals positive Murphy’s sign. Which of the following should be the
first test to determine the patient problem?
a. Rontgent plain film
b. Ultrasonography
c. CT scan
d. ERCP
e. Secretin test

16. A 35-year-old man comes with yellowish color in his skin. He feels fine, his medical history
is unremarkable, and he takes no medications. Lately, he has noted dark urine and pale
stools. The patient’s vital signs are normal, but he is clearly jaundiced. His abdomen is
nontender and no organomegaly. Complete blood count and electrolyte and amylase
levels are normal. An abdominal ultrasound shows multiple small gallstones in the
gallbladder but none in the common bile duct. The common bile duct, however, is dilated.
Which of the following should be the next step in diagnosing this patient?
a. CT
b. ERCP
c. Transhepatic cholangiography
d. Repeat ultrasound
e. Cholescintigraphy

17. A 38-year-old man with debilitating Crohn disease who has undergone a 40 cm ileal
resection presents for evaluation. He has progressive non-bloody diarrhea since his
surgery 9 months ago, which is worse in the evening. He denies having abdominal pain,
nausea or vomiting, fevers, chills, or sweats. He reports no recent travel, camping, or use
of antibiotics. The physical examination is unremarkable. Chemistries show modest
hypokalemia and a mild non–anion gap acidosis. Fecal fat quantitative analysis reveals
minimal steatorrhea.. Which therapy is most likely to help this patient?
a. Loperamide
b. Cholestyramine
c. Probiotics
d. High-protein, low-fat diet
e. Somatostatin

18. Which of the following is the underlying mechanism of the diarrhea?


a. Malabsorption
b. Hypermotility
c. Hypersecretion
d. Normal flora disturbance
e. Hyperosmolarity

19. Mrs. Kartini, an Indonesian 30 year-old-lady, came to Puskesmas because she became
fatigue easily and felt unwell since a month ago. She was found compos mentis with
normal vital signs. Body weight: 50 kg, body height: 150 cm. Based on her body mass index
calculation, her nutritional status was determined in what classification?
a. underweight
b. normal range
c. pre-obese
d. overweight
e. obese I

20. If then Mrs. Kartini was diagnosed as a medical patient but did not need to be
hospitalised, how much was her energy requirement?
a. 1000 kcal
b. 1230 kcal
c. 1530 kcal
d. 2000 kcal
e. 2230 kcal
f. Key: C

21. 3. In terms of macronutrient requirement, how much calorie did Mrs. Kartini need for
carbohydrates?
a. 15-20 % of her total energy requirement
b. 25-30 % of her total energy requirement
c. 35-40% of her total energy requirement
d. 45-50% of her total energy requirement
e. 55-60% of her total energy requirement
22. Meri, a 3 year old girl, came to the Puskesmas with ulcer on her left leg. Through the
history taking it was revealed that Meri had not been well fed in this past year since her
father lost his job. From her physical examination it was found that her BMI (body mass
index) was below -2 SD (standard deviation) for girls of her age. She was fully alert but
looked pale with a face appearance like a thin old person. Her ribs were very prominent
but her both legs were oedema. Based on this information, Meri was likely to be classified
in which condition below?
a. marasmus
b. kwashiorkor
c. marasmic-kwashiorkor
d. energy malnutrition
e. protein malnutrition

23. 5. Osti, a 2-year-old boy came to the outpatient clinic with chief complaint that he had
bow legs with O shape. Osti just moved to Jakarta a month ago. Before that, he lived in
Norway since he was born. According to his parents, Osti had a partial gastrectomy when
he was 1 year old due to a traffic accident.
Physical examination: Head: skull: flattening of the posterior skull
1. Teeth: none
2. Spine: mild scoliosis

Based on this information, what was the underlying problem that caused Osti’s condition?
a. vitamin D deficiency
b. vitamin E deficiency
c. vitamin B deficiency
d. vitamin K deficiency
e. mineral Ca deficiency

24. A 21 years old woman, a professional model, came to the outpatient clinic because she
had difficulties to concentrate, memory problem and also bad smell mouth odour. Since
the last month she reduced her food intake to lose some weight. She hardly ate and only
drank plain water or ate an apple when she was really hungry. She often had headache
and got faint easily. She also felt anxious and had sleep problem.

What was the underlying mechanism that caused the problems?


a. The energy source come mostly from carbohydrate
b. The energy source come mostly from protein
c. The energy source come mostly from fat
d. The energy source come mostly from water
e. The energy source come mostly from vitamin
25. A 10 year old boy, was too hungry to chew his rice properly before swallowing. His
grandmother warned him, he was not going to get any energy from his food. While his
mother did not agree with his eating style she was of the opinion that he would still
benefit from his hastily swallowed food. What is the reason of his grandmother opinion?

A Rice is a good source of fiber


B Amylose in rice is easily digested
C Saliva swallowed with rice contains amylase activity
D Pancreas excretes amylase and other digestive enzymes
E Intestinal glands excrete amylase more active than salivary amylase

26. A 40 year old man has been treated with omeprazole for esophageal reflux.
While this inactivates a proton pump involved in the secretion of gastric acid,
chronic use may stimulate gastrin secretion. What is the consequence of continuous gastrin secretion?

A Inhibition of pepsinogen secretion


B Inhibition of pepsinogen activation
C Inhibition of vitamin B12 absorption
D Stimulation of acid secretion into gastric lumen
E Stimulation of water secretion from mucosal wall

27. Rina, a 20 year old student, had difficulty to concentrate while attending class because of
abdominal pain. They were very strong and continuous though not in a regular way. She
stays in a rented room and takes her meals when it is convenient to go out. After visiting
the student health center she found out that the pains were caused by

A Irregular menstrual cycle


B Spasms of the gastro-intestinal tract
C Decreased motility of the gastro-intestinal tract
D Excessive fluid secretion into gastric lumen
E Deficiency of pancreatic enzymes

28. Fearing of overweight, Lila age 28, has for some time taken an inhibitor of α-amylase
obtained from red kidney beans. She was losing weight because of maldigestion.What is
the effect of the red kidney beans on extraction product on digestion mechanism?

A insufficient Secretion of gastric juice


B insufficient Secretion of pancreatic juice
C insufficient Digestion of oligosaccharides
D insufficient Digestion of glycogen
E insufficient Hydrolysis of maltose

29. A 4 year old girl, brought to a clinic with chief complain diarrhea since this morning after
she had a glass of milk. Every time she had diary product she had diarrhea. What
substance may be deficient in this patient?
A Gastric protease
B Pancreatic amylase
C Lingual lipase
D Intestinal sucrase
E Intestinal lactase

30. Pak Saman works as a street sweeper. One day he suffer from a disease, which caused
massive loss of fluid and electrolytes. He was taken
to a clinic and received intravenous administration of

A KCl D KCl and glucose


B HCl E NaCl and glucose
C Glucose

31. After consulting a doctor and results of lab tests came out, Hadi was told that his
abdominal pain was caused by pancreatic insufficiency resulting in steatorrhea.
The lab test indicated an increase of

A Blood cholesterol D Urinary ketonebodies


B Blood bilirubin E Urinary glucose
C Fecal fat

32. Sari drastically limited her salt intake for fear of hypertension. In doing so she
actually suppressed a cotransport mechanism in the intestinal wall which relies on
Na+ for the absorption of

A Monoacylglycerol D Glycerol
B Free fatty acid E Glucose
C Vitamin D

33. At a ladies lunch gathering, Ibu Rina told her audience not to consider cholesterol
as absolutely harmful to the body and to be drastically avoided. She was actually.
referring to some of the benefits of cholesterol. Which of the following substance was synthesized by
cholesterol?

A Bile acids D Cholestyramine


B Bile pigments E Chylomicrons
C Cholecystokinin

34. A 10 years old boy, suffering from starvation was taken to a clinic
for treatment. He still had good appetite but the doctor advised the nurse.
not to give him a high protein diet for a while. He is still in need of energy
from dietary carbohydrates. The reasoning of the doctor’s advise is

A Pepsin activity is still very low


B Trypsinogen is not yet sufficiently secreted
C Short peptides can easily penetrate intestinal mucosa
D Amino acid absorption is linked to active Na +, K+-pump
E Glucose is better absorbed in absence of intestinal peptides

35. M, a 38-year-old businessman goes to see his doctor with complaints of watery, foul-
smelling diarrhea and flatulence for the past 3 weeks. He reports feeling fatigued since his
return from Kalimantan 2 months previously, and has suffered abdominal cramping and
intermittent loose, nonbloody stools since then. Microscopic examination in stool
specimen shows pear-shaped flagellated trophozoites. Which one of the following
organisms is the MOST likely cause?
a. Entamoeba coli
b. Entamoeba histolytica
c. C. Balantidium coli
d. Giardia lamblia
e. Cryptosporidium parvum

36. Of the following, which one is the treatment of choice on the upper clinical case?
a. co-trimoxazole
b. ketoconazole
c. Metronidazole
d. thiabendazole
e. Mebendazole

37. A 25 years old female comes to the clinic with chief complaint of diarrhea form that 1
week duration. The diarrhea is watery with blood and mucous. Microscopic examination
shows protozoan with pseudopodia. Which of the following is the most likely diagnosis?
a. minuta stage of Entamoeba histolytica
b. vegetative stage of Entamoeba histolytica
c. C. cystic stage of Entamoeba histolytica
d. vegetative stage of Entamoeba coli
e. cystic stage of Entamoeba coli

38. Of the following, which one is the most likely mode by which this disease (upper clinical
case) as acquired?
a. ingestion of the larvae on the organism in contaminated food
b. ingestion of the egg on the organism in contaminated food
c. ingestion of the cystic on the organism in contaminated food
d. drinking unboiled water supply
e. penetration of the skin by the organism while walking bare-footed

39. A 30 years old male works on pig farm. He presents with complaints of bloody and mucous
diarrhea of more than 2 months duration. Laboratory examination of the stools show 60-
70 micron cystic parasites with macronulei. Which of the following is the most likely
diagnosis?
a. Entamoeba coli
b. Entamoeba histolytica
c. Balantidium coli
d. Giardia lamblia
e. Cryptosporidium parvum

40. A 24-years old man has been able to eat well for 2 weeks because of vomiting, abdominal
pain, and 3 months watery diarrhea, no blood or mucous. His laboratory results show :
HIV positive, and CD4 is 200/ul. Microscopic stool examination with modified acid-fast
(Kiyoun) stain shows 4 μm oocysts. Which of the following is the most likely diagnosis?
a. Blastocystis hominis
b. Entamoeba histolytica
c. C. Balantidium coli
d. .Giardia lamblia
e. Cryptosporidium parvum

41. A 40-years old man goes to see his doctor with complaints of significant weight loss,
flatulence, and steatorrhoeic. He reports feeling fatigued 3 months previously, and has
suffered abdominal cramping. Which of the following is the initial investigations should be
made ?
a. An ELISA test
b. duodenal aspirate for microscopy
c. microscopic examination of stools
d. X-ray films
e. CT-scan
42. While examining a tissue of the GI tract, you notice a stratified squamous non-keratinizing
epithelium lining the lumen, with a thick muscularis mucosa layer beneath it. Which of the
following part of the GI tract are you examining?
a. Oral cavity
b. Esophagus
c. Stomach
d. Duodenum
e. Anal canal

43. While examining a tissue of the GI tract, you notice a group of mucous glands in the
submucosa layer and many intestinal villi. Which of the following part of the GI tract are
you examining?
a. Duodenum
b. Jejunum
c. Ileum
d. Colon
e. Appendix

44. While examining a tissue of the GI tract, you notice in the lamina propria layer many
straight, tubular, branched glands, consisting of several kinds of cells. One kind of them
are large, acidophilic cells. Which of the following cells are you describing?
a. Stem cells
b. Mucous neck cells
c. Parietal cells
d. Chief (zymogenic) cells
e. Enteroendocrine cells

45. While examining a tissue of the GI tract, you notice many lymphoid nodules, surrounding a
relative small lumen. No intestinal villi is seen. Which of the following part of the GI tract
are you examining?
a. Duodenum
b. Jejunum
c. Ileum
d. Colon
e. Appendix
46. While examining a tissue of the GI tract, you notice many intestinal villi. On one side of the
wall you notice aggregates of lymphoid nodules in the lamina propria and submucosa,
opposite the mesenterium.Which of the following part of the GI tract are you examining?
a. Duodenum
b. Jejunum
c. Ileum
d. Colon
e. Appendix

47. To optimize its absorptive function, the small intestines is differently equipt than the large
intestines. Which of the following structures are found in the small intestines, but not in
the large intestines?
a. Crypts of Lieberkühn
b. Glands of Lieberkühn
c. Simple columnar epithelium
d. Intestinal villi
e. Auerbach plexus

48. While examining a tissue of the GI tract, you notice a gradual transition of the epithelial
lining from stratified squamous epithelium to simple columnar epithelium.
a. Which of the following part of the GI tract are you examining?
b. Transition from duodenum to jejunum
c. Transition from esophagus to gaster
d. Transition from gaster to duodenum
e. Transition from rectum to anus
f. Transition from ileum to colon

49. The liver is an exocrine and endocrine gland. To fulfill its exocrine function, what
microscopic structure is seen in a microscopic section of the liver?
a. Hepatic artery branch
b. Portal vein branch
c. Bile duct
d. Central vein
e. Portal triad

50. Both hepatic lobule and liver acinus are found in the liver.Which of the following feature is
true for liver acinus, but not for hepatic lobule?
a. Hexagonal
b. A structural unit
c. Involved in transporting of blood
d. 3 or 4 portal areas at its periphery
e. Involved in transporting of bile

51. At the periphery of a hepatic lobule are several portal areas. A portal area is the place
where blood enter and bile leave the liver lobule.
a. Which of the following structure is not found in the portal area?
b. Hepatic artery branch
c. Portal vein branch
d. Bile duct
e. Central vein
f. Lymphatic vessel

52. When viewing a liver section with high magnification, you notice a thin slit between the
hepatocytes and the endothelium of the sinusoids.
Which of the following name is correct for this slit?
a. Bile canaliculus
b. Bile ductule
c. Hering’s canal
d. Perisinusoidal canal
e. Space of Disse

SOAL 53 DAN 54
A 17 year old male presented jaundice and dark yellow urine. He also complained of nausea, low-grade
fever, loss of appetite, and right hypochondrial pain. Of relevance in his history was the fact that
he had recently eat in a food stall with his friends, and his friends also complained same
symptoms . His physician sent a blood sample for analysis.

53. Which of the following serological profile has detectable in this patient?
a. Anti-HAV IgM
b. Anti-HBV IgM
c. Anti-HCV IgM
d. Anti-HDV IgM
e. Anti-HEV IgM

54. Which of the following modes of transmission is most likely to be implicated in this case?
a. Fecal-oral route
b. Blood-borne
c. Close contacts
d. Vector bite
e. Air-borne

SOAL 55 DAN 56
A 22 year old male had a history of having multiple sexual partners over the past year presented with
jaundice. He admitted because having pale stool and dark urine. Clinical examination revealed a
slightly enlarge liver with some right hypochondrial tenderness. The serological profile was
negative for IgG-HBs and positive for IgG-HBc, HBsAg.

55. Which of the following best describe his illness? Dr cucu


a. Early acute HBV infection
b. Chronic HBV infection
c. Window period
d. Vaccine-type response
e. Never infected

56. The patient had a girlfriend, who was seronegative. Which of the following preventing
measures could protect this girl?
a. Advice abstinence from sexual intercourse until he become asymptomatic
b. Give both a course of α-interferon
c. Administer hepatitis B immunoglobulin (HBIG)
d. Administer a single dose of HBIG and the HBV vaccine
e. Give both a course of ribavirin

Soal 57 dan 58
A female tourist developed gastroenteritis while visiting small town in Indonesia and tried some
Indonesian traditional food. The onset of the disease is abrupt with abdominal cramps and watery
diarrhea. She had no fever or nausea or vomiting. The symptoms have resolved within 24 hour
and no subsequent recurrences. They report the disease to district public health office. The
investigation found that one of the food products eaten by this tourist was contaminated by
suspected pathogens.

57. Which food is the most likely source of this infection ?


a. Rendang
b. Gado-gado
c. Fried rice
d. Pepes
e. Coconut ice

58. What is the suspected pathogen may cause the disease above ?
a. Salmonella typhi
b. Shigella dysenteriae
c. Enterohemorragic E. coli
d. Staphylococcus aureus
e. Enterotoxigenic E.coli

59. Regarding suspected pathogen mentioned No. 58, what statement below is correct ?
a. Produce Shiga Toxin (shigella)
b. Heavy fecal leukocytes found
c. Colon involved (shigella)
d. Differentiated by Kaufmann-White scheme (salmonela)
e. Common nasal normal flora (aureus)

Approximately 4 hour after eating a meal in restaurant, 2 members of a college student group develop a
sudden onset of nausea, vomiting, dizziness and severe abdominal cramps. Nobody got febrile and
only 1 had diarrhea. Within a day, the symptoms has disappeared and no recurrences.

60. Which organism is most likely responsible for this outbreak ?


a. Salmonella typhi
b. Shigella dysenteriae
c. Enterohemorragic E. coli
d. Staphylococcus aureus
e. Enterotoxigenic E.coli

61. Which of the following is the most important specimen to be collected to confirm
diagnosis ?
a. Blood
b. Urine
c. Food
d. Fecal
e. Vomiting material

62. Which is correct statement about suspected agents above ?


a. Bacteria invade small bowel through M cells
b. Pathogen above evoke producing of B cell to produce immune system response
c. Symptoms caused by toxin named Shiga-like toxin
d. Reheated in room temperature before cooked is a good steps in food preparation
e. Inadequate cold storage before serving providing ideal conditions for bacterial growth
and enterotoxin production

63. A 36 years old woman came to private family doctor complaining fever for 7 days, nausea,
vomiting, anorexia and diarrhea.
Physical examination :
Temperature 39 °C, blood pressure 100/70 mmHg, pulse rate 80 per minute, respiratory rate 20 per
minute. She was moderately ill-appearing, dry and white mucous membrane and tremor tongue,
Hiperactive bowel abdominal sound and mild diffuse tenderness
Laboratory findings :
WBC count of 4500/mm3 , diff count 0/2/13/42/41/2
What organism may cause these symptomps above?
a. Shigella dysenteriae
b. Enteropathogenic E.coli
c. Salmonella enteritidis
d. Vibrio cholerae
e. Campylobacter

64. Microscopic of responsible pathogen’s Gram staining is :


a. Gram-negative rod
b. Gram-positive rod
c. Chain shape Gram-positive coccus
d. Cluster shape Gram-positive coccus
e. Coma shape Gram-negative

65. A 30 years old woman suffer with gastric pain and burning sensation all the time and
relieved by food and antacids. Family doctor ask her to get endoscopic examination and
found ulcer in the surface of gastric mucosa.
The pathogen that may cause the disease above is
a. Enteroaggregative E. Coli
b. Helicobacter pylori
c. Yersinia enterolitica
d. Campylobacter jejuni
e. Vibrio parahaemolyticus

66. The true statement below is


a. Pleimorphic shapea nd microaerophilic Gram negative rod
b. High concentration of urease product expressed allowing this pathogen to generate
ammonium ion that buffer gastric acid
c. May cause colon carcinoma by aplastic changing of colon mucosa
d. Urease breath test is a definitive test to identify the microorganism
e. Antibiotic treatment is not a mandatory in this case

67. Mrs.X, 42 years old, came to emergency department with a mild fever, followed by
malaise, fatigue, nausea and vomiting for 5 days. Acholic stool and darkening urine are
also appeared. Sclera and the skin looked yellow. The liver is palpable 2 cm below the
costal margin, tender and the spleen is enlarged. The patient was unconscious. No prior
history of the disease.What is the mechanism of the darkening urine in the case above?
a. The unconjugated bilirubin filtered by the glomerulus
b. The red blood cells filtered by the glomerulus and appears in the urine
c. It is appears due to inflammation reaction in the renal
d. The unconjugated bilirubin is water soluble so it can appears in the urine
e. The conjugated bilirubin increase in plasma,readily filtered by the glomerulus
68. A 60 years old man, came to hospital with unconscious condition. From his relatives, we
knew that epigastric pain, nausea and heartburn were rare . Some medications ( Antacid
etc ) had been taken but there was no improvement. He has taken aspirin for his heart
disease since 5 years ago. There was no hematemesis and no abnormality on the physical
findings. Endoscopic results showed some erosions and bleeding particularly in the
anthrum.What is the most recommended treatment to prevent this disease?
a. Mucoprotector
b. H-2 receptor blocker
c. COX-2 specific inhibitors
d. Proton Pump Inhibitor
e. Prostaglandin analogue

69. What is the laboratory finding might show?


a. Increased platelet
b. Decreased hemoglobine
c. Decreased white blood count
d. Increased AST
e. Prolonged APTT

70. What substance synthesis is impaired in the case above?


a. Kinin
b. Pepsin
c. Glycoprotein
d. Gastrin
e. Prostaglandin

71. 34 year old patient with dyspepsia and epigastric pain, was admitted to the polyclinic. The
lab test result was Hb 12 mg/dl, WBC 15.000/ul, Platelet 300.000. The doctor suspected
that the patient is suffering from acute gastritis. What further investigation will help the
doctor to confirm his definitive diagnosis?
a. Biopsy
b. Endoscopy
c. Culture of Biopsy tissue
d. Culture of Gastric fluid
e. Blood culture

72. Which of the following organism is the etiology?


a. C. pylori
b. H. pylori
c. C. fetus
d. C. jejuni
e. H. influenzae
73. Which of the following test is the non invasive for this case?
a. Biopsy
b. Detection of IgG
c. PCR
d. Rapid urea test
e. Anatomical pathology test

74. A 20-month-old boy presents with a diarrhea since 3 days before admission. He is having
watery diarrhea for 3-5 times per day with blood and mucous; not malodorous, and are
not bulky or oily. Physical examination: irritable and lethargic. BP 88/50 Pulse 110, RR 32,
temp 38,6 OC. He has sunken eyes and fontanel; decreased tears; dry mucous membranes;
mild delay in elasticity (skin turgor); and delayed capillary refill. His urine output is less
than usual. He is no apparent distress. His breath sounds are equal and clear. His abdomen
is nondistended with positive bowel sounds. There are no masses and no
hepatosplenomegaly. There are no perianal lesions. What is the most likely etiology of his
diarrhea?
a. Infection
b. Drugs
c. Allergy
d. Malabsorption
e. Vitamin deficiency

75. What is your initial management for this patient?


a. Oral rehydration solution 100-200mL every episode of diarrhea
b. Isotonic IV solution 30 ml/kg/0,5 hours, then 70 ml/kg/2,5 hours
c. Isotonic IV solution 100 ml/kg/3 hours
d. Oral rehydration solution 75 ml/kg/3hrs
e. Oral Rehydration ad libitum (as much as patient desired )

76. The laboratory studies that would allow you to make the diagnosis include:
a. stool for fat
b. a stool culture
c. stool examination for parasites
d. a stool ELISA
e. stool for reducing substance and pH
77. A 1-month-old baby presents with a diarrhea since 2 days before admission. He is having
watery diarrhea for 4-6 times per day with no blood no mucous; acid smell, and are not
bulky or oily. Her mother complained that her baby has rash around his buttock.
What substance that is not properly absorbed in the case above?
a. fats
b. proteins
c. carbohydrate
d. vitamins
e. minerals

78. The antibiotic that most often leads to Clostridium difficile infection in pediatric patients is:
a. clindamycin
b. amoxicillin
c. erythromycin
d. cefadroxil
e. ciprofloxacin

79. A 60-year-old woman came to emergency room because of cramping abdominal pain,
vomiting, distended abdomen, and lack of defecation or flatus for the last 2 days.
Abdominal examination elicited abdominal distention and hyperactive of bowel sound.

What is the initial diagnostic test of the case above?


a. colonoscopy
b. abdominal CT-Scan
c. gastroscopy
d. ultrasonography
e. abdominal radiographs

80. The most likely diagnosis for the woman presented is :


a. Pancreatic tumor
b. Gastric carcinoma
c. Perforated appendicitis
d. Colon tumor
e. Rectal polyp

81. A 50-year-old man is admitted with massive bright red rectal bleeding. He recently had a
barium enema that demonstrated no diverticular or mass. Nasogastric suction reveals no
blood but does produce yellow bile. The patient continues to bleed.
What is the next diagnostic step?
a. Repeat barium enema
b. Colonoscopy
c. Upper gastrointestinal series
d. Mesenteric angiography
e. Small bowel follow-through with barium

82. The most likely cause of the bleeding is?


a. Internal hemorrhoid
b. Regional enteritis
c. Duodenal ulcer
d. Intussuception
e. Angiodysplasia

83. A 22-year-old man whose father and older brother died of colon cancer is evaluated for
painless rectal bleeding. A flexible sigmoidoscopy reveals the presence of multiple polyps
throughout the sigmoid colon, with sparing of the rectum.What is the next procedure to
be performed?
a. Repeat sigmoidoscopy and biopsy
b. Barium enema
c. Radionuclide imaging
d. Colonoscopy and biopsy
e. Selective arteriography

84. The most likely diagnosis for the man presented is :


a. Villous adenomas
b. Zollinger-Ellison syndrome
c. Familial adenomatous polyposis
d. Colon cancer
e. Hamartomatous polyps

JAWABAN : E, D, B, E, D, C, D, B, D, C, A, A, E,
85. A 8-year-old girl presents in the emergency department with a 24-hour history of
increasingly severe right lower quadrant abdominal pain. The pain began in the
periumbilical region, which was associated with nausea and vomiting. On physical
examination, she has tenderness in the right lower quadrant with rebound, and there is no
mass. On rectal examination also reveals right-sided tenderness. Low grade fever and mild
leukocytosis are present.The most accurate diagnostic test of the case is :
a. Barium enema
b. Upper gastrointestinal series
c. Ultrasonography D
d. CT scan abdomen with contrast
e. Flat and upright abdominal radiographs

86. What is the most likely diagnosis?


a. Acute diverticulitis
b. Acute appendicitis B
c. Ureteral stone
d. pyelonephritis
e. Intussusception

87. The initial management for this patient should be


a. Broad-spectrum antibiotic therapy
b. Analgesics
c. Observation
d. Fasting and prepare for surgery
e. Diagnostic laparoscopy

JAWABAN : E, D, B, E, D, C, D, B, D, C, A, A, E,

88. The definitive therapy should be


a. Laparotomy
b. Drainage
c. Appendectomy
d. Ureterolithotomy
e. Diverticular resection

JAWABAN : E, D, B, E, D, C, D, B, D, C, A, A, E,
89. A 45-year-old man presents in the emergency room with a history of acute onset of severe
epigastric pain radiating toward the back. The pain began several hours after consuming
fatty meal. The patient has no significant medical history and denies any previous surgery.
On physical examination, the patient has marked epigastric tenderness with guarding and
hypoactive bowel sounds. The patient’s amylase level is 2500 units.What is the most likely
cause of this patient’s pain?
a. Gallstones
b. GERD
c. Duodenal ulcer A
d. Nephrolithiasis
e. Peptic ulcer
90. Initial treatment for this patient should be :
a. Nothing by mouth, insertion a nasogastric tube, fluid resuscitation, insertion of a Foley
catheter and antibiotics
b. Bile duct exploration
c. Endoscopic retrograde cholangiopancreatography (ERCP)
d. Emergency cholecystectomy and intraoperative cholangiogram
e. Pancreatic debridement and drainage

91. What is the possible origin for referred pain to right shoulder
a. Kidney
b. Ureter
c. Pancreas
d. Duodenum
e. Gallbladder
92. What is the possible diagnosis of left lower quadrant abdominal pain
a. Gastritis
b. Pancreatitis
c. Herpes zoster
d. Diverticulitis
e. Perforated ulcer

93. 70.A 45- year old man was admitted to the emergency department complaining of severe
pain in the right lower quadrant of the anterior abdominal wall. He had repeatedly
vomited, and his temperature and pulse rate were elevated. On examination, the muscles
of the lower part of the anterior abdominal wall in the right lower quadrant showed
tenderness at the MC Burney point. You are at surgery for removal, but the organ is not
visible. What position of this organ is likely to be?
a. Paracecal
b. Paracolic
c. Retrocecal C
d. Iliocecal
e. Retrocolic
94. A surgery procedure was performed and you encounter an artery and vein in the
superficial fascia of the lower abdominal wall. Which vessel are most likely the superficial
of the lower abdominal wall?
a. Superficial epigastria artery
b. Circumflex iliac artery
c. External iliac artery A
d. Superior epigastria artery
e. Inferior epigastria artery

95. 73.During surgery, you must incise the anterior rectus sheath between the xiphoid process
and the umbilicus.In this region, which of the following muscle derives the rectus sheath?

a. External abdominal oblique only


b. Internal abdominal oblique only
c. External and Internal abdominal oblique C
d. Internal oblique and transverses abdominal
e. Transversus abdominal only

96. 74.A 42-year old male has a 6 months history of epigastric pain and is constant in nature
especially after meals. His passed tarry stool (black stool) over the previous 2 moths.
Laboratory of stool’s test showed occult blood.What organ is the most likely to be
affected?
a. jejunum
b. gaster
c. ileum B
d. appendix
e. gallbladder

97. 75.A 45-year-old obese woman complaining of indigestion was admitted to hospital for
investigation. She had past history of gallstone and transient attack of jaundice. Large
gallstone have been known to erode through the posterior wall of the gallbladder and
enter the intestinal tractWhich part of the intestinal tract is likely to initially contain the
stone?
a. Sigmoid colon
b. Descending colon
c. Transverse colon C
d. Ascending colon
e. Jejunum

98. 76.A 55 year old man with a long history of duodenal ulcer was seen in emergency room
after vomiting blood and exhibiting severe hypovolemic shock. Assuming the ulcer had
perforated the posterior wall of the first part of duodenum, which artery is most likely to
have been eroded?
a. Splenic artery
b. Right gastric artery
c. Gastroduodenal artery C
d. Right gastroepiploica artery
e. Proper Hepatic artery

99. Bonda 11 months old boy. He was well until four months ago when his mother stopped
breast-feeding and began giving him cow’s milk with other food. Since then Bondan has
had two episodes of diarrhea, the current one having begun three days ago. Bondan still
takes cow’s milk but his mother has reduced his intake of solid food since the diarrhea
begun. There has been no blood in the stool. Bondan weighs 8 kg. What should you do in
this case ?
a. Give Bondan antibiotic for possible infection
b. Prescribe a special lactose-free formula and have the mother give this in place of the
cow’s milk.
c. Tell the mother to give Bondan sweetened fruit drinks or tea, whichs he likes, so that he
receives enough fluid

d. Advise the mother to dilute Bondan’s milk with an equal amount of water, oral
rehydration solution and to increase other energy-rich foods in his diet, if not improved
he should be referred to hospital
e. Tell the mother to take Bondan to a hospital right away
Key : D

100. A 50 year old man was healthy until he developed vomitting five days ago. He said that he
did not eat as much as usual. He was feeling amount of food in his epigastric area since
one month ago. He had history of rheumatoid arthritis. Mrs. RANI was go to the hospital,
and get some medication. Which of the following factors is not correlate to the incidence
and severity of the symptom?
a. Chili and sour food
b. Usually becoming late in breakfast time
c. Drink black coffee and tea in the morning C
d. Using NSAID for long time
e. Helycobacter pylori infection

101. Kurmin, a 10 year old boy, bring by her mother to policinic because he had fever and
jaundice for five days. He also suffered for vomiting for about 5 times per day. When the
doctor examine him, the doctor note that he is lethargic, his eyes are yellow, she has
hepatomegalyi. Kurmin appear to be undernourished. Based on these findings, what
conclusions would you draw about Kurmin’s condition, which one is not correct ?
a. HAV is transmitted commonly most via the fecal-oral route. Epidemics of HAV infection
may be explained by person-to-person contact, such as occurs at institutions, or by
exposure to a common source, such as consumption of contaminated water or food.
b. The incubation period of HAV is 15-45 days (average 4 wk). The virus is excreted in stool
during the first few weeks of infection, prior to the onset of symptoms. Acute hepatitis A
is more severe and has higher mortality in adults than in children.
c. Typical cases of acute HAV infection are marked by several weeks of malaise, anorexia,
nausea, vomiting, and elevated aminotransferase levels. Jaundice develops in more
severe cases.
d. Acute infection is documented by the presence of immunoglobulin M (IgM) anti-HAV,
which disappears several months after the initial infection.
e. Hospitalization is needed for all patients of hepatitis A.

102. LENA, aged 3 years old, is brought to the Puskesmas because of bloody defecation, which
began three days ago. At first the stools were only loose, but yesterday his mother saw
blood in them. The mother believes LENA has no fever. Which of the following statement
must ask to confirm gastrointestinal bleeding in children?
a. Anorectal disorders, fissures, and distal polyps produce bright red blood
b. Melena rather than bright red blood per rectum is usually a sign of bleeding that comes
from a source proximal to the ligament of Treitz.
c. C. Massive upper GI bleeding, however, can produce bright red blood per rectum if GI
transit time is rapid.

d. Familial history or nonsteroidal anti-inflammatory drug (NSAID) use may suggest ulcer
disease. Ingested substances, such as NSAIDs, steroids, and foreign bodies, can irritate
the gastric mucosa enough to cause blood to be mixed with the vomitus.
e. Recent jaundice, easy bruising, and changes in stool color may not important signal for GI
bleeding in liver disease.

103. Five year old boy with right lower abdominal pain, has been brought to the Hospital by her
mother. He has been assessed and found to have some signs of appendicitis. What is not
appropriate statement about this case?
a. Appendicitis is the most common surgical emergency of children
b. Duration of symptoms is associated with ruptured (perforation), and delay of surgical
treatment
c. Delay in surgical definitive management, increase the risk of perforation and hospital
length of stay.

d. Young children had the lowest incidence of acute appendicitis, but greater risk of
complication than teenage.
e. The small children can communicate their symptoms as well as adult.

104.You are assessing a patient with ascites by having the patient assume the knee chest
position. After a few minute, you do percussion on the umbilical area. This technique for
assessing ascites is called:
a. Shifting dullness
b. Fluid wave
c. Puddle sign C
d. Cullen’s sign
e. Psoas sign

105. Rota is 9 months old and was healthy until he developed diarrhea a half weeks ago. The
episode began with stools that were loose and sometimes watery. The frequency of
diarrhea was about three times/day. Each time of diarrhea is about a half aqua cup. During
the last few days Rota’s mother has noted “acid smell” in his stool. Rota has vomited
several times in the past days.
a. Allergic diarrhea
b. Acute dysentery
c. Acute diarrhea C
d. Chronic diarrhea
e. Persistent diarrhea with blood

106. Rota is 6 months old and was healthy until he developed diarrhea two days ago. Rota’s
mother says that he does not eat as much as usual. He was weaned from breast milk to
cow’s milk at age of 3 months. Rota’s immunization is complete. Rota was bring to the
Puskesmas and get Oralit. Which of the following factors is not correlate to the incidence
and severity of diarrhea in young children ?
a. Washing hands after defecation and before preparing food
b. Exclusive breast feeding for the first 4-6 months of life
c. Bathing the child frequently C
d. Using infant bottles
e. Immunization against measles

107. Lakto, a 1 year old boy, bring by her mother to emergency room because he had diarrhea
for five days. The frequency was 10 times per day. Each time was about a half aqua cup.
He also suffered for vomiting for about 5 times per day. When the doctor examine him,
the doctor note that he is lethargic, his eyes are sunken, she has no tears when she cries,
his mouth is dry, and he didn’t drink eagerly. His skin pinch goes back slowly (more than 2
seconds). Lacto appear to be undernourished. Based on these findings, what conclusions
would you draw about Lacto’s condition and how she should be treated ?
a. Lakto has some dehydration and should be treated according to treatment plan A
b. Lakto has severe dehydration and should be treated according to treatment plan B
c. Lakto has severe dehydration and should be given by the intra venous fluid
d. Lakto has some dehydration and should be treated by giving infussion
e. Lakto has some dehydration and should be treated by giving oral rehydration solution
Key : C

108. Lakto, aged 13 months, is brought to the Puskesmas because of diarrhea, which began
three days ago. At first the stools were only loose, but yesterday his mother saw blood in
them and smelly. The mother believes Lakto has a fever. He has also vomited three time
per day. When you examine Lakto, you note that he is alert, irritable and restless. His eyes
are not sunken, he has tears when he cries, his mouth is moist, he will drink some water.
His skin goes back quickly. His temperature is 38 C. Which of the following statement is
correct?
a. Lakto has some dehydration and should be treated according to treatment plan B
b. Lakto has some dehydration and should be infused
c. Lakto has no signs of dehydration, give the oral rehydration solution and should be
treated for dysentery

d. Lakto has no signs of dehydration and just give the oral rehydration solution
e. Lakto has some dehydration and should be treated for dysentery
109. A mother has brought her 2 years old daughter, Rena to the Puskesmas. Rena has been
assessed and found to have some signs of dehydration. She weighs 10 kg. While at the
Puskesmas, her mother has given her 600 ml of Renalit/ORS solution within 3 hours. After
4 hours, Rena still has some signs of dehydration, but is improving. Assuming that the
mother can stay at the Puskesmas, what shoul be done next?
a. Stop the Renalit and give 500 ml KAEN3B solution intravenously during the next 3 hours
b. Repeat plan B, giving 750 ml of Renalit in the next 4 hours and start to feed the child as
described in plan A.

c. Pass an nasogastric tube and give 1000 mL ORS solution by the NGT in the next 4 hours
d. Suggest the mother that Rena can go home
e. Give the mother the antibiotic for diarrhea

110. Rena, 1n 16 month old baby with diarrhea, has been brought to the Puskesmas by his
grandmother. She weighs 10 kg. She has been assessed and found to have some signs of
dehydration. The grandmother must leave soon to catch the last bus, it is too far for her to
walk home. What is the unappropriate suggestions?
a. Give the grandmother 750 ml of ORS solution and show her how to give it to Rena
during the next 4 hours.
b. Explain to the grandmother how Rena should be fed when they return home
c. Give the grandmother packets of ORS for use in treating Rena at home during the next
2 days, after he has been rehydrated

d. Give the child an antibiotic to treat his infection


e. Explain to the grandmother when she must go back to the Puskesmas by assessing
Rena’s condition

111. A 38 years old man comes to the emergency with fatigue and abdominal swelling. For
several months he has noticed that his abdomen has becam larger and his skin turned in
yellow. On examination, his palm shows redness, his abdomen is markedly distended and
a fluid wave is present. What organ is likely to be affected?
a. Gall bladder
b. Pancreas
c. Liver
d. Portal vein
e. Stomach

112. A 44 year old man complains of discomfort in his right upper thigh over the past 6
months. He works in the garden department of a home improvement centre. On physical
examination, there is a bulge at the right inguinal area that is not reducible. What is the
most likely diagnosis?
a. Femoral Hernia
b. Lymphadentis inguinalis
c. Congenital Inguinal Hernia

d. Acquired Inguinal Hernia


e. Incarcerated Hernia

113. Today, Adi a student of Faculty of Medicine Universitas Pelita Harapan, want to go to his
classmate Ani. Ani said that her mother has jaundice. So she call Adi to have suggestion
about her mother.According to Ani, several week ago his mother suffered abdominal colic
for several days. Now the colic is not come again, but since two days ago, her mother got
jundice. So, Adi think about what happened to Ani’s mother. He suggest Ani to make a
laboratory test for her mother, to know what kind of substance that may increase its
concentration.Adi think that Ani’s mother jaundice is caused by:
a. Hepatitis
b. hepatic cirrhosis
c. portal obstruction

d. hemolytic disease
e. bile duct obstruction

114. Substance that may increase in concentration is:


a. globulin
b. albumin
c. free bilirubin

d. cholecystokinin
e. conjugated bilirubin

115. Ani’s mother stool may indicate:


a. normal
b. melena
c. diarrhea

d. very fatty
e. very hard

116. The substance that may responsible to the golden yellow colour of bile is:
a. albumin
b. globulin
c. cholecystokinin
d. alkaline phosphatase
e. bilirubin and biliverdin

Before going back to his dormitory, Adi goes to Cafe Murah to have lunch. During waiting for his meal,
he sits in a table. Suddenly, he smell something very attractive flavor, so his saliva is coming out
from his saliva glands.

117. The digestive enzyme that we may can find in Adi’s saliva is:
a. maltase
b. protease
c. peptidase

d. -amylase
e. none of the above.

118. The pH of Adi’s saliva in his condition now is about:


a. 4
b. 6
c. 8

d. 10
e. 12

119. The conditioned stimulus that may stimulate the secretion of Adi’s saliva is:
a. food in the mouth
b. seeing a tasty food
c. thinking a tasty food

d. smelling a tasty food


e. sitting on the restaurant table to wait a meal

A 17 year old healthy boy is having breakfast. He masticate the food slowly, to give attention to his
food in his mouth. He feels that the saliva is still coming out from the saliva’s glands, and
make mastication is going more gentle, the taste of the food more tasty. Then he swallow
the bolus. He can manage the action of swallowing.

120. What is the first reflex in swallowing food?


a. glottic closure
b. inhibition of respiration
c. relaxation of lower esophageal sphincter
d. contraction of esophageal muscles
e. contraction of pharyngeal muscles

121. What Reflex allow the food to enter the stomach?


a. glottic closure
b. inhibition of respiration
c. relaxation of lower esophageal sphincter (LES)

d. peristaltic ring contraction of esophageal muscles


e. contraction of pharyngeal muscles that pushes bolus into the esophagus

122. Adi think about the regulation of the GI tract function. It must be exist some systems that
may regulate the function of the GI tract, so we need not worry and to think about it.
According to the anatomy lessons, there are some plexuses and nervous systems that
regulate the function of GI tract systems so that they can work automatically and
independent to our mind. Adi remember that one of the system is called the enteric
nervous system. Which one is constituted what Adi remember:
a. hypogastric plexus
b. sympathetic nervous system
c. parasympathetic nervous system

d. the myenteric and submucous plexus


e. the preganglionic fibers of sympathetic nervous system

123. Adi also remember that the enteric nervous system is concerned with the function of:
a. deglutition
b. defecation
c. hepatic portal system

d. viscerosensory system
e. motility and secretion of the intestines

124. A 15 year old boy is having a cup of coffee for the first time. After 30 minutes, he feel
nausea and then vomiting. During vomiting, he feels the muscles of his abdominal wall
contract forcefully and then the content of his stomach is push back to his mouth and to
the outside. He thinks that between stomach and the mouth during normal conditions
there must be something that prevent the content of the stomach to flow back to the
mouth.Which organ plays an important role in the prevention of gastric reflux?
a. glottis
b. diaphragm
c. sphincter pylorus
d. lower esophageal sphincter
e. peristaltic ring contraction of esophageal muscles
125. 16. In the normal condition, during what condition it relaxes?
a. resting
b. coughing
c. expiration
d. inspiration
e. swallowing

126. It is 13:00 o’clock in the afternoon. Adi feel a mild pain in the epigastric region as usual if
he does not take a meal in this time. Since 7:00 in the morning, he does not eat
something.What is the cause of mild pain?
a. angina
b. bile colic
c. muscle cramp

d. duodenal colic
e. hunger contraction

GASTROINTESTINAL BLOCK
PEMBUAT SOAL: dr. DARWIN FERDINAN SALIM

127. A 60-year-old male, come to the Siloam Hospital with a chief complain of bloody vomiting.
The blood was red and fresh. The patient also tells that his stool was dark. At physical
examination, there are palmar eritema, spider neavi, and ascites at the abdomen. Where
do you think is the blood came from?
a. Rupture of Esophageal veins
b. Gastric perforation
c. Rupture of Mesentrica Superior veins
d. Rupture of Mesentrica Inferior veins
e. Rupture of Superior cava vein

128. A 60-year-old male, come to the Siloam Hospital with a chief complain of bloody vomiting.
The blood was red and fresh. The patient also tells that his stool was dark. At physical
examination, there are palmar eritema, spider neavi, and ascites at the abdomen. The USG
show a nodul on the right lobule of the hepar. Which of the following test that can help
you diagnose?
a. CA 19-9
b. -feto protein
c. PSA
d. CA-153
e. Seromoeba

129. A 46-year-old Female, come to the clinic, with a chief complain of severe pain in the upper
right abdomen. the pain is not show up all the time, and it is spreading to the upper
middle abdomen. the patient also complains of nausea, vomiting and fever. On the
physical examination, her weight is 80kgs, height 160cm, on the abdomen, Murphy’s sign
+. What is the most likely diagnosis?
a. Perforating of gastric ulcers
b. Acute Pancreatitis
c. Gastroduodentitis
d. Acute Cholesistitis
e. Acute Pyelonefritis

130. A 40-year-old male traveler from Germany, come to the hospital with a chief complain of
diarrhea, 10 times per day since last night. His stool was yellow without any blood or
mucus. He said that he had a glass of cendol in the morning and at night he gets diarrhea.
On the physical examination, the mental status compos mentis, BP: 110/80 mmHg, Pulse:
84x/min, temp: 37,5C.What do you think that caused this patient diarrhea?
a. Salmonella
b. E. Coli
c. Virus Norwalk
d. Shigella
e. Amoeba

131. A mother bring her 3-month-old baby to the hospital with a chief complain of diarrhea 6
times a day, since 2 days ago. She also tells that her baby was vomiting, and became
irritable. She also tells that she gave her baby powder cow milk (2 table spoon with 30cc of
water) for 3 days, before the baby get diarrhea. On the physical examination, the baby
weight is 7 kg, Temp : 36.4 OC, pulse: 110x/min, irritable, crying with small amount of tears,
skin turgor slightly decrease. What is the type of diarrhea that the baby suffer?
a. Secretoric diarrhea
b. Traveller’s diarrhea
c. Exudative diarrhea
d. Osmotic diarrhea
e. Food poisoning diarrhea

132. As a surgeon performing appendectomy, you encounter an appendicular artery.Which


vessel is directly giving branch to the appedicular artery?
a. ileocolica artery
b. colica sinistra artery
c. jejunal artery
d. ileal artery
e. colica dextra artery
133. 2. What is the origin of the vessel above?
a. mesenterica inferior a.
b. mesenterica superior a.
c. coeliaca a.
d. superior rectalis a.
e. middle colic a

134. During surgery, you must incise the anterior rectus sheath between the xiphoid process
and the umbilicus.In this region, which muscle derives the rectus sheath?
a. External abdominal oblique only
b. Internal abdominal oblique only
c. External and Internal abdominal oblique
d. Internal oblique and transverses abdominis
e. Transversus abdominis only

135. A 44 year old man complains of discomfort in his right upper thigh over the past 6 months.
He works in the garden department of a home improvement centre. On physical
examination, there is a bulge above inguinal ligament located medial to inferior epigastirc
artery.What is the most likely diagnosis?
a. Femoral Hernia
b. Lymphadentis inguinalis
c. Direct Inguinal Hernia
d. Indirect Inguinal Hernia
e. Inguinal Lipoma

136. A 55 year old woman has a bulge in the groin since 3 months ago. She is an over weight
woman with 100 kg BW. She was diagnosed with femoral hernia.Where is the location of
femoral hernia is:
a. above inguinal ligament medial to hypogastric artery
b. above inguinal ligament lateral to hypogastric artery
c. below inguinal ligament in femoral triangle
d. below inguinal ligament in inguinal triangle

137. You are examining the liver during a surgical procedure. The inferior caval vein is located in
between which of two anatomical lobes?
a. Quadrate and caudate lobes
b. Caudate and left lobes
c. Right and quadrate lobes
d. Left and quadrate lobes
e. Right and caudate lobes
138. Esophagus, stomach, duodenum, jejunum, ileum and colon are derivatives of GIT. All of
them differentiate from ……………
a. foregut, midgut and hindgut
b. only foregut and midgut
c. only midgut and hindgut
d. primitive gut
e. endoderm

139. There are two pancreatic duct bring the enzymes produced by pancreas.To what structure
the enzyme is drained?
a. Stomach
b. Duodenum pars superior
c. Duodenum pars descenden
d. Jejunum
e. Ileum

140. A 25 year old man with typhoid fever and acute abdominal pain. The surgeon is examining
the small intestine during a surgical procedure. As a clerk in the surgery department, you
have to know the difference between jejunum and ileum What is the most alteration
between jejunum and ileum?
a. Diameter jejunum smaller than ileum
b. The wall of jejunum thicker than ileum
c. Vascularity of jejunum less than ileum
d. Color of jejunum pale that ileum
e. Plica circularis of jejunum shorter than ileum

141. A 69 year old man with abdominal mass underwent laparatomy. You are examining the
large intestine during the surgical procedure. To avoid cutting the vessel, you must know
the topography of the intestine. What structure is supplied by the inferior mesenteric?
a. Colon descenden
b. Caudate ascenden
c. Colon transversum
d. Jejunum
e. Ileum

142. Meri, a 3 year old girl, came to the Puskesmas with ulcer on her left leg. Through the
history taking it was revealed that Meri had not been well fed in this past year since her
father lost his job. From her physical examination it was found that her BMI (body mass
index) was below -2 SD (standard deviation) for girls of her age. She was fully alert but
looked pale with a face appearance like a thin old person. Her ribs were very prominent
but her both legs were oedema.
Based on this information, Meri was likely to be classified in which condition below?

a. marasmus
b. kwashiorkor
c. marasmic-kwashiorkor
d. energy malnutrition
e. protein malnutrition

143. While examining a tissue of the GI tract, you notice many intestinal villi. On one side of the
wall you notice aggregates of lymphoid nodules in the lamina propria and submucosa,
opposite the mesenterium. Which of the following part of the GI tract are you examining?
a. Duodenum
b. Jejunum
c. Ileum
d. Colon
e. Appendix

144. The liver is an exocrine and endocrine gland. To fulfill its exocrine function, what
microscopic structure is seen in a microscopic section of the liver?
a. Hepatic artery branch
b. Portal vein branch
c. Bile duct
d. Central vein
e. Portal triad

145. M, a 38-year-old businessman goes to see his doctor with complaints of watery, foul-
smelling diarrhoea and flatulence for the past 3 weeks. He reports feeling fatigued since
his return from Kalimantan 2 months previously, and has suffered abdominal cramping
and intermittent loose, nonbloody stools since then. Microscopic examination in stool
specimen shows pear-shaped flagellated trophozoites. Which one of the following
organisms is the MOST likely cause?
a. Entamoeba coli
b. Entamoeba histolytica
c. Balantidium coli
d. D.Giardia lamblia
e. E. Cryptosporidium parvum

146. A 30 years old male works on pig farm. He presents with complaints of bloody and mucous
diarrhea of more than 2 months duration. Laboratory examination of the stools show 60-
70 micron cystic parasites with macronuclei. Which of the following is the most likely
diagnosis?
a. Entamoeba coli
b. Entamoeba histolytica
c. Balantidium coli
d. Giardia lamblia
e. Cryptosporidium parvum

147. A 24-years old man has been able to eat well for 2 weeks because of vomiting, abdominal
pain, and 3 months watery diarrhea, no blood or mucous. His laboratory results show :
HIV positive, and CD4 is 200/ul. Microscopic stool examination with modified acid-fast
(Kiyoun) stain shows 4 μm oocysts. Which of the following is the most likely diagnosis?
a. Blastocystis hominis
b. Entamoeba histolytica
c. Balantidium coli
d. Giardia lamblia
e. Cryptosporidium parvum

148. A 17 year old male presented jaundice and dark yellow urine. He also complained of
nausea, low-grade fever, loss of appetite, and right hypochondrial pain. Of relevance in his
history was the fact that he had recently eat in a food stall with his friends, and his friends
also complained same symptoms . His physician sent a blood sample for analysis. Which
of the following serological profile has detectable in this patient?
a. Anti-HAV IgM
b. Anti-HBV IgM
c. Anti-HCV IgM
d. Anti-HDV IgM
e. Anti-HEV IgM

149. A 34-year-old woman has the sudden onset of severe abdominal pain. On physical
examination she is afebrile. The pain is centered in the mid-epigastric region, though there
is marked diffuse tenderness in all quadrants. Laboratory studies show her serum amylase
is 410 U/L and lipase is 610 U/L.
Which of the following laboratory test findings is most likely to be present in this woman?
a. Hypoproteinemia
b. Positive urea breath test
c. Hyperglycemia
d. Elevated sweat chloride
e. Positive serology for HBsAg

150. A 32 year old woman comes to the emergency room with severe epigastric and right
upper quadrant pain 3 hours after eating gulai in Minang restaurant. The pain occasionally
radiates to her right flank and right back. She has slight fever and her abdomen is tender
to palpation in the right upper quadrant. Her lab tests are as follows:
ALT: 44 U/l (<30 )
AST: 25 U/l (<30 )
Total bilirubin: 1,3 mg/dl (0-1)
Alkaline phosphatase: 100 U/l (38-126)
White blood 14500 (6000-10000)
Amylase: 67 U/l (53-123)
Lipase: 55 U/l (10-150)
What is the most likely explanation for this patient’s symptoms?
a. Acute cholecystitis
b. Acute peptic ulcer
c. Acute pancreatitis
d. Acute appendicitis
e. Acute hepatitis B

151. A 52-year-old man with peptic ulcer disease has been on drug therapy for 3 months. While
on this regimen, he noticed changes in his bowel habits, increasing headaches, dizziness,
skin rashes, loss of libido, and gynecomastia. Which drug is the most likely responsible for
these effects?
a. cimetidine
b. famotidine
c. misoprostol
d. omeprazole
e. ranitidine

152. A 62-year-old man with chief complain nausea,vomiting, and epigastric pain. He was given
stomach medication, three hour later, he had dyskinesia and rigidity (extra pyramidal
symptoms). Which of the following drug is the most appropriate that can cause this
situation?
A. domperidone
B. metoclopramide
C. ondansetrone
D. omeprazole
E. sucralfate

153. A 51-year-old man comes to your private practice with a history of mid epigastric upper
abdominal pain that diminished by eating or taking antacid tables. He drinks 2-3 glasses of
beer every night, also taking 2-3 tablets of aspirin frequently for his stress related
headache. PE reveals no evidence for blood in the stool. You advised him to replace the
aspirin with acetaminophen and stop his drinking of beer; you give him a prescription of
famotidine 20 mg bid. Which of the following is the mechanism of action of famotidine?
a. Eradication of H.pylori
b. Increase of motility
c. Inhibit acid secretion
d. Neutralized gastric acid
e. Protect gastric mucous

154. A 35-year-old man comes with yellowish color in his skin. He feels fine, his medical history
is unremarkable, and he takes no medications. Lately, he has noted dark urine and pale
stools. The patient’s vital signs are normal, but he is clearly jaundiced. His abdomen is
nontender and no organomegaly. Complete blood count and electrolyte and amylase
levels are normal. An abdominal ultrasound shows multiple small gallstones in the
gallbladder but none in the common bile duct. The common bile duct, however, is dilated.
Which of the following should be the next step in diagnosing this patient?
A. CT
B. ERCP
C. Transhepatic cholangiography
D. Repeat ultrasound
E. Cholescintigraphy

162.A 45-year-old man presents in the emergency room with a history of acute onset of severe
epigastric pain radiating toward the back. The pain began several hours after consuming fatty
meal. The patient has no significant medical history and denies any previous surgery. On
physical examination, the patient has marked epigastric tenderness with guarding and
hypoactive bowel sounds. The patient’s amylase level is 2500 units. What is the most likely
cause of this patient’s pain?
a. Gallstones
b. GERD
c. Duodenal ulcer
d. Nephrolithiasis
e. Peptic ulcer

163. A 60-year-old male, come to the Siloam Hospital with a chief complain of bloody vomiting. The
blood was red and fresh. The patient also tells that his stool was dark. At physical examination,
there are palmar eritema, spider neavi, and ascites at the abdomen. The USG show a nodul on
the right lobule of the hepar.
Which of the following test that can help you diagnose?
a. CA 19-9
b. -feto protein
c. PSA
d. CA-153
e. Seromoeba

164. A 46-year-old Female, come to the clinic, with a chief complain of severe pain in the upper right
abdomen. the pain is not show up all the time, and it is spreading to the upper middle
abdomen. the patient also complains of nausea, vomiting and fever. On the physical
examination, her weight is 80kgs, height 160cm, on the abdomen, Murphy’s sign (+). What is
the most likely diagnosis?
a. Perforating of gastric ulcers
b. Acute Pancreatitis
c. Gastroduodentitis
d. Acute Cholesistitis
e. Acute Pyelonefritis

165. A mother bring her 3-month-old baby to the hospital with a chief complain of diarrhea 6 times a
day, since 2 days ago. She also tells that her baby was vomiting, and became irritable. She also
tells that she gave her baby powder cow milk (2 table spoon with 30cc of water) for 3 days,
before the baby get diarrhea. On the physical examination, the baby weight is 7 kg, Temp :
36.4OC, pulse: 110x/min, irritable, crying with small amount of tears, skin turgor slightly
decrease.
What is the type of diarrhea that the baby suffer?

a. Secretoric diarrhea
b. Traveller’s diarrhea
c. Exudative diarrhea
d. Osmotic diarrhea
e. Food poisoning diarrhea

166. Mrs.X, 42 years old, came to emergency department with a mild fever, followed by malaise,
fatigue, nausea and vomiting for 5 days. Acholic stool and darkening urine are also appeared.
Sclera and the skin looked yellow. The liver is palpable 2 cm below the costal margin, tender and
the spleen is enlarged. The patient was unconscious. No prior history of the disease. What is the
mechanism of the darkening urine in the case above?
a. The unconjugated bilirubin filtered by the glomerulus
b. The red blood cells filtered by the glomerulus and appears in the urine
c. It is appears due to inflammation reaction in the renal
d. The unconjugated bilirubin is water soluble so it can appears in the urine
e. The conjugated bilirubin increase in plasma,readily filtered by the glomerulus

A 60 years old man, came to hospital with unconscious condition. From his relatives, we knew
that epigastric pain, nausea and heartburn were rare . Some medications ( Antacid etc ) had
been taken but there was no improvement. He has taken aspirin for his heart disease since 5
years ago. There was no hematemesis and no abnormality on the physical findings. Endoscopic
results showed some erosions and bleeding particularly in the anthrum.

167. What is the laboratory finding might show?


a. Increased platelet
b. Decreased hemoglobine
c. Decreased white blood count
d. Increased AST
e. Prolonged APTT

168. What substance synthesis is impaired in the case above?


a. Kinin
b. Pepsin
c. Glycoprotein
d. Gastrin
e. Prostaglandin

169. A 69 year old man with abdominal mass underwent laparatomy. You are examining the large
intestine during the surgical procedure. To avoid cutting the vessel, you must know the
topography of the intestine. What structure is supplied by the inferior mesenteric?

a. Colon descenden
b. Caudate ascenden
c. Colon transversum
d. Jejunum
e. Ileum

170. A 45- year old man was admitted to the emergency department complaining of severe pain in
the right lower quadrant of the anterior abdominal wall. He had repeatedly vomited, and his
temperature and pulse rate were elevated. On examination, the muscles of the lower part of
the anterior abdominal wall in the right lower quadrant showed tenderness at the MC Burney
point. You are at surgery for removal, but the organ is not visible.
What position of this organ is likely to be?

a. Paracecal
b. Paracolic
c. Retrocecal

d. Iliocecal
e. Retrocolic

171.Fearing of overweight, a 28-year-old woman has for some time taken an inhibitor of α-amylase
obtained from red kidney beans. She was losing weight because of maldigestion. Effect of red
kidney beans extraction product on digestion mechanism was insufficiency to carry out:
a. Secretion of gastric juice
b. Secretion of pancreatic juice
c. Digestion of oligosaccharides
d. Digestion of glycogen
e. Hydrolysis of maltose

172.A 21 years old woman, a college student, presents with complaint of abdominal discomfort in the
upper abdomen since 2 weeks ago, everyday but it’s not all day long. It feels like her stomach
bloated. Unpleasant abdominal fullness after a normal size meal. The pain centered especially at
epigastric region, without any radiation. The pain is not associated with change of position, or
rest. Sometimes the meal could relieve the abdominal discomfort. She said she never
experience heartburn, nausea, or vomits. She denies any dysphagia, odynophagia, fever, or
cough. She has no problem with urination and defecation. She had a regular period; the last
period was 3 weeks ago. Endoscopy reveals no abnormalities. What is the most likely
diagnosis?
a. Ulcer like dyspepsia
b. Reflux like dyspepsia
c. Dismotility like dyspepsia
d. Unspecific dyspepsia
e. Organic ulcer dyspepsia

173.An 18-year-old man comes to the clinic with fever since 7 days ago. He also complains
constipation, nausea, and vomiting. He brings laboratory test which shown that he got positive
result for Widal test. On the basis of your histologic knowledge which part of the GI tract was
the primary location of infection of the disease?
a. Duodenum
b. Jejunum
c. Ileum
d. Colon
e. Appendix

174.A 20-month-old boy presents with a diarrhea since 3 days before admission. He is having watery
diarrhea for 3-5 times per day with blood and mucous; not malodorous, and is not bulky or oily.
Physical examination: irritable and lethargic. BP 88/50 Pulse 110, RR 32, temp 38,6 OC. He has
sunken eyes and fontanel; decreased tears; dry mucous membranes; mild delay in elasticity
(skin turgor); and delayed capillary refill. His urine output is less than usual. He is no apparent
distress. His breath sounds are equal and clear. His abdomen is nondistended with positive
bowel sounds. There are no masses and no hepatosplenomegaly. There are no perianal lesions.
What is your initial management for this patient?
a. Oral rehydration solution 100-200mL every episode of diarrhea
b. Isotonic IV solution 30 ml/kg/0,5 hours, then 70 ml/kg/2,5 hours
c. Isotonic IV solution 100 ml/kg/3 hours
d. Oral rehydration solution 75 ml/kg/3hrs
e. Oral Rehydration ad libitum (as much as patient desired )

175.A 43-year-old woman came to the emergency room with colicky abdominal pain on the right
upper quadrant of the abdomen. On the sonography examination the doctor found a stone in
the duct which drains directly to gallbladder. Which of the following structure was seen on the
examination?
a. Santorin’s duct
b. Wirsung’s duct
c. Common bile duct
d. Cystic duct
e. Hepatic duct

176.A 45-year-old woman complained to her physician with discomfort and abdominal pain localized
to the right upper quadrant. An abdominal USG showed a multiple diffuse nodules in liver.
Biopsy showed tumor cells that have more prominent nucleoli, coarser chromatin and Mallory’s
hyaline. This finding is typical for:
a. Hepatocellular adenoma
b. Hepatocellular carcinoma
c. Cavernous Hemangioma
d. Cholangio sarcoma
e. Hamartoma

177.A 17-year-old girl intravenous drug user with history of nausea, vomiting and passage of dark
urine was brought into ER with high fever by her mother. Physical examination revealed icteric
sclera, mild jaundice, enlargement liver and several fresh-healed track marks on her arms. A
liver biopsy showed hydrophic degeneration of hepatocytes. Which of the following liver
diseases is most likely to present?
a. Liver cirrhosis
b. Acute hepatitis A infection
c. Acute hepatitis B infection
d. Choleducholithiasis
e. Fatty steatosis

178.A 55-year-old woman whom you examine for the first time feels well. You do a complete physical
examination, which is normal except for a few very small palpable and moveable, non-tender
nodes in both cervical chains. However, her laboratory studies show the following hepatitis B
virus profile: HBsAg (-), anti-HBs (+), low levels of IgG anti-HBc, HBeAg (-), and anti-HBe (+).
What is the most likely diagnosis in this patient?
a. Acute HBV infection, high infectivity
b. Late-acute HBV, low infectivity
c. Recovered from HBV infection
d. Chronic HBV infection, high infectivity
e. Immunization with HBsAg vaccine

179.A 40-years-old woman comes to the Emergency Department with upper abdominal pain that lasts
for several hours. The pain gradually increases in severity, and localizes to the epigastrium and
right hypochondrium, with radiation to the shoulder area. She also experiences nausea,
vomiting, anorexia, and low grade fever. The findings on physical examination are mild icteric
sclerae, epigastric and hypochondrium tenderness, inspiratory arrest on palpation of the right
upper quadrant. What structure is the most involved from this patient?
a. Liver
b. Pancreas
c. Duodenum
d. Gall bladder
e. Gaster

180.A 45-years-old woman presents with colic abdominal pain suddenly and persist with severe
intensity for 3 hours. She feels steady ache and fullness in the epigastrium and right upper
quadrant with radiation to the right scapula. Colic is precipitated by eating a fatty meal. What is
the most appropriate laboratory examination?
a. Alkaline Phosphatase
b. SGOT and SGPT
c. Bilirubin
d. Amilase
e. Ca 19-9

181.A 43-year-old woman came to a clinic with complains of yellowish color on her eyes and skin. Five
days ago she suffered right upper abdominal colicky pain for several days especially after eating
fatty food. Now the colic is gone for two days. Which substance that most likely to be increase
in the serum?
a. Unconjugated bilirubin
b. Albumin
c. Globulin
d. Cholecystokinin
e. Conjugated bilirubin

182.A 60-year-old male, come to the Siloam Hospital with a chief complain of bloody vomiting. The
blood was red and fresh. The patient also tells that his stool was dark. At physical examination,
there are palmar eritema, spider naevi, and ascites at the abdomen. The USG show a nodul on
the right lobule of the hepar. Which of the following test that can help you diagnose?
a. CA 19-9
b. -feto protein
c. PSA
d. CA-153
e. CEA

183.At a ladies lunch gathering, Mrs. Rina told her audience not to consider cholesterol as absolutely
harmful to the body and to be drastically avoided. She was actually referring to one of the
benefits of cholesterol which is a precursor for the synthesis of
a. Bile acid
b. Bile pigments
c. Cholecystokinin
d. Cholestyramine
e. Chylomicrons

184.Mrs. Kartini, an Indonesian 30 year-old-lady, came to Puskesmas because she became fatigue
easily and felt unwell since a month ago. She was found compos mentis with normal vital signs.
Body weight: 50 kg, body height: 150 cm. Based on her body mass index calculation, her
nutritional status was determined in what classification?
a. underweight
b. normal range
c. pre-obese
d. overweight
e. obese I

185.Meri, a 3-year-old girl, came to the Puskesmas with ulcer on her left leg. Through the history
taking it was revealed that Meri had not been well fed in this past year since her father lost his
job. From her physical examination it was found that her BMI (body mass index) was below -2
SD (standard deviation) for girls of her age. She was fully alert but looked pale with a face
appearance like a thin old person. Her ribs were very prominent but her both legs were
oedema. Based on this information, Meri was likely to be classified in which condition below?
a. marasmus
b. kwashiorkor
c. marasmic-kwashiorkor
d. energy malnutrition
e. protein malnutrition

186.A 42-year-old alcoholic man came into your clinic with severe epigastric pain. The pain comes
abruptly 30 minutes after meal, he also vomit 2 times on that period. On physical examination
you’ve got blood pressure 80/60 mmHg, heart rate 132 x/min, respiratory rate 28 x/min,
temperature 37,4 OC, tenderness and guarding on left upper quadrant, and diminished bowel
sound. Lab result returns with serum amylase 1000 (53-153) and serum lipase 4500 (10-150).
RBS 250 mg/dL (<200 mg/dL). What is the basic pathologic mechanism of the condition above?
a. Infection
b. Malignancy
c. Autodigestion
d. Obstruction
e. Perforation

187.A 10-month-old boy brought to the emergency department by his mother with abdominal pain
and vomiting in the recent 2 hour. The boy was brought to a pediatrician yesterday because of
diarrhea; the last stool was like red-currant-jelly and has stopped since the boy suffers the pain.
On the examination there is distended and tender abdomen, a palpable mass with sausage-like-
appearance on right side of abdomen, and increased bowel sound.
Which of the following findings may be seen on the plain abdominal photos?
a. Air distribution in the sigmoid colon
b. Free-air under the diaphragm
c. Enlargement of the liver
d. Dilated small bowel with air-fluid level
e. Thickening of the wall of appendix

188.Where is the most possible anatomical location of this pathologic condition?


a. Transverse colon
b. Duodenum
c. Liver
d. Ileocecal junction
e. Appendix

189.A 17-year-old male presents to the emergency department with 2 days diarrhea. There is neither
abdominal pain nor blood in the stool. On examination he is dehydrated and recognized to have
infectious diarrhea due to Vibrio cholerae. What is the pathophysiology of his diarrhea?
a. The toxin migrates through the wall to paralyze the muscular layer
b. The toxin activates intracell enzyme and secretes chloride ions
c. The toxin causes necrosis of the secretory glands of the colon
d. The toxin causes necrosis and slough of the intestinal mucosa
e. The toxin causes irritable bowel syndrome

190.A 31-year-old man has continuous symptoms of dyspepsia. He takes large amounts of antacids
during and following his meals. Despite this he develops an ulcer. The most likely cause for this
in increased secretion of which hormone?
a. Gastrin
b. Secretin
c. CCK
d. Somatostatin
e. Pancreatic bicarbonate

MCQ Gastrointestinal Block 2009


23 March 2009

191. Mrs. Kartini, an Indonesian 30 year-old-lady, came to Puskesmas because she became fatigue
easily and felt unwell since a month ago. She was found compos mentis with normal vital signs.
Body weight: 50 kg, body height: 150 cm.
If then Mrs. Kartini was diagnosed as a medical patient but did not need to be hospitalised, how
much was her energy requirement?
a. 1000 kcal
b. 1230 kcal
c. 1530 kcal
d. 2000 kcal
e. 2230 kcal

192. Meri, a 3 year old girl, came to the Puskesmas with ulcer on her left leg. Through the history
taking it was revealed that Meri had not been well fed in this past year since her father lost his
job. From her physical examination it was found that her BMI (body mass index) was below -2
SD (standard deviation) for girls of her age. She was fully alert but looked pale with a face
appearance like a thin old person. Her ribs were very prominent but her both legs were
oedema.
Based on this information, Meri was likely to be classified in which condition below?

a. marasmus
b. kwashiorkor
c. marasmic-kwashiorkor
d. energy malnutrition
e. protein malnutrition

193.Osti, a 2-year-old boy came to the outpatient clinic with chief complaint that he had bow legs
with O shape. Osti just moved to Jakarta a month ago. Before that, he lived in Norway since he
was born. According to his parents, Osti had a partial gastrectomy when he was 1 year old due
to a traffic accident. Physical examination: Head: skull: flattening of the posterior skull; Teeth:
none; Spine: mild scoliosis.Based on this information, what was the underlying problem that
caused Osti’s condition?

a. vitamin D deficiency
b. vitamin E deficiency
c. vitamin B deficiency
d. vitamin K deficiency
e. mineral Ca deficiency
194.While examining a tissue of the GI tract, you notice a stratified squamous non keratinizing
epithelium lining the lumen, with a thick muscularis mucosa layer beneath it. Which of the
following part of the GI tract are you examining?

a. Oral cavity
b. Esophagus
c. Stomach
d. Duodenum
e. Anal canal

195.While examining a tissue of the GI tract, you notice in the lamina propria layer many straight,
tubular, branched glands, consisting of several kinds of cells. One kind of them are large,
acidophilic cells.Which of the following cells are you describing?

a. Stem cells
b. Mucous neck cells
c. Parietal cells
d. Chief (zymogenic) cells
e. Enteroendocrine cells

196.While examining a tissue of the GI tract, you notice many lymphoid nodules, surrounding a
relative small lumen. No intestinal villi is seen.Which of the following part of the GI tract are you
examining?

a. Duodenum
b. Jejunum
c. Ileum
d. Colon
e. Appendix

197.While examining a tissue of the GI tract, you notice many intestinal villi. On one side of the wall
you notice aggregates of lymphoid nodules in the lamina propria and submucosa, opposite the
mesenterium.
Which of the following part of the GI tract are you examining?

a. Duodenum
b. Jejunum
c. Ileum
d. Colon
e. Appendix
198. While examining a tissue of the GI tract, you notice a gradual transition of the epithelial lining
from stratified squamous epithelium to simple columnar epithelium.
Which of the following part of the GI tract are you examining?

a. Transition from duodenum to jejunum


b. Transition from esophagus to gaster
c. Transition from gaster to duodenum
d. Transition from rectum to anus
e. Transition from ileum to colon
199. The liver is an exocrine and endocrine gland. To fulfill its exocrine function, what microscopic
structure is seen in a microscopic section of the liver?
a. Hepatic artery branch
b. Portal vein branch
c. Bile duct
d. Central vein
e. Portal triad
200. M, a 38-year-old businessman goes to see his doctor with complaints of watery, foul-smelling
diarrhoea and flatulence for the past 3 weeks. He reports feeling fatigued since his return from
Kalimantan 2 months previously, and has suffered abdominal cramping and intermittent loose,
nonbloody stools since then. Microscopic examination in stool specimen shows pear-shaped
flagellated trophozoites.
Which one of the following organisms is the MOST likely cause?
a. Entamoeba coli
b. Entamoeba histolytica
c. Balantidium coli

D.Giardia lamblia
E. Cryptosporidium parvum

201. A 25 years old female comes to the clinic with chief complain of diarrhea for 1 week. The
diarrhea is watery with blood and mucous. Microscopic examination shows protozoa with
pseudopodia.

a. Which of the following is the most likely diagnosis?


a. minuta stage of Entamoeba histolytica
b. vegetative stage of Entamoeba histolytica
c. cystic stage of Entamoeba histolytica

d. vegetative stage of Entamoeba coli


e. cystic stage of Entamoeba coli

b. Of the following, which one is the most likely mode by which this disease is acquired?
a. ingestion of the larvae on the organism in contaminated food
b. ingestion of the egg on the organism in contaminated food
c. ingestion of the cystic on the organism in contaminated food

d. drinking unboiled water supply


e. penetration of the skin by the organism while walking bare-
footed
202.A 30 years old male works on pig farm. He presents with complaints of bloody and mucous
diarrhea of more than 2 months duration. Laboratory examination of the stools show 60-70
micron cystic parasites with macronuclei. Which of the following is the most likely diagnosis?
a. Entamoeba coli
b. Entamoeba histolytica
c. Balantidium coli

D.Giardia lamblia
E. Cryptosporidium parvum
203. A 24-years old man has been able to eat well for 2 weeks because of vomiting, abdominal pain,
and 3 months watery diarrhea, no blood or mucous. His laboratory results show : HIV positive,
and CD4 is 200/ul. Microscopic stool examination with modified acid-fast (Kiyoun) stain shows 4
μm oocysts.
204. Which of the following is the most likely diagnosis?
a. Blastocystis hominis
b. Entamoeba histolytica
c. Balantidium coli
d. D.Giardia lamblia
e. E. Cryptosporidium parvum

205. A 17 year old male presented jaundice and dark yellow urine. He also complained of nausea,
low-grade fever, loss of appetite, and right hypochondrial pain. Of relevance in his history was
the fact that he had recently eat in a food stall with his friends, and his friends also complained
same symptoms . His physician sent a blood sample for analysis.

206. Which of the following serological profile has detectable in this patient?
a. Anti-HAV IgM
b. Anti-HBV IgM
c. Anti-HCV IgM
d. Anti-HDV IgM
e. Anti-HEV IgM

207. Which of the following modes of transmission is most likely to be implicated in this case?
a. Fecal-oral route
b. Blood-borne
c. Close contacts
d. Vector bite
e. Air-borne

208. A 22 year old male had a history of having multiple sexual partners over the past year presented
with jaundice. He admitted because having pale stool and dark urine. Clinical examination
revealed a slightly enlarge liver with some right hypochondrial tenderness. The serological
profile was negative for IgG-HBs and positive for IgG-HBc, HBsAg. The patient had a girlfriend,
who was seronegative. Which of the following preventing measures could protect his girlfriend?
a. Advice abstinence from sexual intercourse until he become asymptomatic
b. Give both a course of α-interferon
c. Administer hepatitis B immunoglobulin (HBIG)
d. Administer a single dose of HBIG and the HBV vaccine
e. Give both a course of ribavirin

209. 34 year old patient with dyspepsia and epigastric pain, was admitted to the polyclinic. The lab
test result was Hb 12 mg/dl, WBC 15.000/ul, Platelet 300.000. The doctor suspected that the
patient is suffering from acute gastritis.
What further investigation will help the doctor to confirm his definitive diagnosis?
a. Biopsy
b. Endoscopy
c. Culture of Biopsy tissue
d. Culture of Gastric fluid
e. Blood culture

210. Fearing of overweight, Lila age 28, has for some time taken an inhibitor of α-amylase obtained
from red kidney beans. She was losing weight because of maldigestion.
What is the effect of the red kidney beans on extraction product on digestion mechanism?
a. A insufficient Secretion of gastric juice
b. B insufficient Secretion of pancreatic juice
c. C insufficient Digestion of oligosaccharides
d. D insufficient Digestion of glycogen
e. E insufficient Hydrolysis of maltose

211. A 4 year old girl, brought to a clinic with chief complain diarrhea since this morning after she
had a glass of milk. Every time she had diary product she had diarrhea.
What substance may be deficient in this patient?
a. A Gastric protease
b. B Pancreatic amylase
c. C Lingual lipase
d. D Intestinal sucrase
e. E Intestinal lactase

212. At a ladies lunch gathering, Ibu Rina told her audience not to consider cholesterol as absolutely
harmful to the body and to be drastically avoided. She was actually referring to some of the
benefits of cholesterol.
Which of the following substance was synthesized by cholesterol?
A Bile acids D Cholestyramine
B Bile pigments E Chylomicrons
C Cholecystokinin

213. A 13 year-old boy is having a bloody diarrhea 5 times a day for 2 days. He vomits 3 times. He
felt pain in his anus while he passes his stool. He also has fever
What mechanism occur in his bowel when passing the stool?
a. Longitudinal muscle relaxation in receiving segment
b. Circular muscle contraction in receiving segment
c. Longitudinal muscle contraction in propulsive segment
d. Circular muscle contraction in propulsive segment

e. Circular muscle relaxation in propulsive segment


214. A 34-year-old woman has the sudden onset of severe abdominal pain. On physical examination
she is afebrile. The pain is centered in the mid-epigastric region, though there is marked diffuse
tenderness in all quadrants. Laboratory studies show her serum amylase is 410 U/L and lipase is
610 U/L.
Which of the following laboratory test findings is most likely to be present in this woman?
a. A Hypoproteinemia
b. B Positive urea breath test
c. C Hyperglycemia
d. D Elevated sweat chloride
e. E Positive serology for HBsAg

215. A 31-year-old woman has a 10 year history of intermittent, bloody diarrhea. She has no other
major medical problems. On physical examination there are no lesions palpable on digital rectal
examination, but a stool sample is positive for occult blood
This patient is at greatest risk for development of which of the following conditions?
a. A Pancreatitis
b. B Diverticulitis
c. C Cholangitis
d. D Appendicitis
e. E Perirectal fistula

216. A 32 year old woman comes to the emergency room with severe epigastric and right upper
quadrant pain 3 hours after eating gulai in Minang restaurant. The pain occasionally radiates to
her right flank and right back. She has slight fever and her abdomen is tender to palpation in the
right upper quadrant. Her lab tests are as follows:
ALT: 44 U/l (<30 ) AST: 25 U/l (<30 )
Total bilirubin: 1,3 mg/dl (0-1) Alkaline phosphatase: 100 U/l (38-126)
White blood 14500 (6000-10000) Amylase: 67 U/l (53-123)
Lipase: 55 U/l (10-150)

What is the most likely explanation for this patient’s symptoms?


a. Acute cholecystitis
b. Acute peptic ulcer
c. Acute pancreatitis

d. Acute appendicitis
e. Acute hepatitis B

217. A 52-year-old man with peptic ulcer disease has been on drug therapy for 3 months. While on
this regimen, he noticed changes in his bowel habits, increasing headaches, dizziness, skin
rashes, loss of libido, and gynecomastia.
a. Which drug is the most likely responsible for these effects?

b. cimetidine
c. famotidine
d. misoprostol
e. omeprazole
f. ranitidine
218.A 62-year-old man with chief complain nausea,vomiting, and epigastric pain. He was given
stomach medication, three hour later, he had dyskinesia and rigidity (extra pyramidal
symptoms).
a. Which of the following drug is the most appropriate that can cause this situation?
A. domperidone
B. metoclopramide
C. ondansetrone
D. omeprazole
E. sucralfate

219. A 48-year-old man presents with a complaint of chronic no bloody diarrhea and right lower
quadrant pain with a palpable mass and tenderness. He states that this "flare-up" is one of the
worst he has ever experienced. Radiographic exam reveals evidence of ulceration, stricturing,
and fistula development of the colon and small bowel.
Which of the following drugs would be the most appropriate for treating this patient?
A. Diphenoxylate
B. Loperamide
C. Hyoscyamine
D. Mesalamine
E. Prednisone

220. A 70-year-old man with a history of atrial fibrillation is started on an oral anticoagulant. His
prothrombin time is monitored on a regular basis. A few months into his therapy, he begins
treatment for a duodenal ulcer and he develops symptoms of a bleeding diathesis.
a. Which of the following ulcer medications is most likely responsible for this change in his
hemostatic status?
A. Cimetidine
B. Famotidine
C. Misoprostol
D. Rabeprazole
E. Ranitidine

221. A 51-year-old man comes to your private practice with a history of mid epigastric upper
abdominal pain that diminished by eating or taking antacid tables. He drinks 2-3 glasses of beer
every night, also taking 2-3 tablets of aspirin frequently for his stress related headache. PE
reveals no evidence for blood in the stool. You advised him to replace the aspirin with
acetaminophen and stop his drinking of beer; you give him a prescription of famotidine 20 mg
bid.
Which of the following is the mechanism of action of famotidine?

a. Eradication of H.pylori
b. Increase of motility
c. Inhibit acid secretion
d. Neutralized gastric acid
e. Protect gastric mucous
222. A 48-year-old woman recently diagnosed with an adenocarcinoma. The oncologist schedules
her for radiation and chemotherapy. Because nausea and vomiting are common complications,
you recommend antiemetic premedication with dexamethasone.
Which of the following antiemetic drugs would be the most appropriate for this condition?

a. Chlorpromazine
b. Dimenhydrinate
c. Ondansetron
d. Prometazine
e. Scopolamine

223. A 24 year old woman comes to your private practice because of experiencing heartburn after
meals, especially when reclining on her sectional at home. Her symptoms are typically relieved
by antacids. In addition to an H2 blockers or PPI drugs, what additional agent could help treat
her esophagitis by coating necrotic tissue with a protective barrier, and thereby promote
healing of the ulcer?
a. Attapulgite
b. Calcium carbonate
c. Loperamide
d. Mg-Al hydroxide
e. Sucralfate

224. A 18-year-old man presents with intermittent cramp abdominal pain, no bloody diarrhea, and
weight loss of 5 kg over 6 months. The diarrhea, wake him from sleep. On a few occasions, he
has had fevers, nausea, and vomiting. The patient describes his work as being stressful. PE:
normal vital signs. He has an oral aphthous ulcer (sariawan) and poorly localized lower
abdominal to mid abdominal tenderness without peritoneal signs. Anal and rectal examinations
are normal, and a stool guaiac test (for detecting occult blood) is negative. Stool leukocytes are
present. The Hb 11.5 and Ht is 34%. Results of sigmoidoscopy are normal. Which of the
following is the most likely diagnosis for this patient?
a. Acute appendicitis
b. Crohn disease
c. Colon cancer
d. Irritable bowel syndrome
e. Ulcerative colitis

225. A 50-year-old man comes to your clinic complaining of acute severe upper abdominal pain that
radiates to his back and worsens with meals. He has a long history of binge drinking. He notes
that lately he has been losing weight and that his stools have been loose. Physical examination
reveals positive Murphy’s sign. Which of the following should be the first test to determine the
patient problem?
a. Rontgent plain film
b. Ultrasonography
c. CT scan
d. ERCP
e. Secretin test

226. A 35-year-old man comes with yellowish color in his skin. He feels fine, his medical history is
unremarkable, and he takes no medications. Lately, he has noted dark urine and pale stools.
The patient’s vital signs are normal, but he is clearly jaundiced. His abdomen is nontender and
no organomegaly. Complete blood count and electrolyte and amylase levels are normal. An
abdominal ultrasound shows multiple small gallstones in the gallbladder but none in the
common bile duct. The common bile duct, however, is dilated. Which of the following should be
the next step in diagnosing this patient?
a. CT
b. ERCP
c. Transhepatic cholangiography
d. Repeat ultrasound
e. Cholescintigraphy

227. A 38-year-old man with debilitating Crohn disease who has undergone a 40 cm ileal resection
presents for evaluation. He has progressive non-bloody diarrhea since his surgery 9 months ago,
which is worse in the evening. He denies having abdominal pain, nausea or vomiting, fevers,
chills, or sweats. He reports no recent travel, camping, or use of antibiotics. The physical
examination is unremarkable. Chemistries show modest hypokalemia and a mild non–anion gap
acidosis. Fecal fat quantitative analysis reveals minimal steatorrhea.
Which of the following is the underlying mechanism of the diarrhea?

a. Malabsorption
b. Hypermotility
c. Hypersecretion
d. Normal flora disturbance
e. Hyperosmolarity

A 20-month-old boy presents with a diarrhea since 3 days before admission. He is having watery
diarrhea for 3-5 times per day with blood and mucous; not malodorous, and are not bulky or
oily. Physical examination: irritable and lethargic. BP 88/50 Pulse 110, RR 32, temp 38,6 OC. He
has sunken eyes and fontanel; decreased tears; dry mucous membranes; mild delay in elasticity
(skin turgor); and delayed capillary refill. His urine output is less than usual. He is no apparent
distress. His breath sounds are equal and clear. His abdomen is nondistended with positive
bowel sounds. There are no masses and no hepatosplenomegaly. There are no perianal lesions.

228. What is the most likely etiology of his diarrhea?


a. Infection
b. Drugs
c. Allergy
d. Malabsorption
e. Vitamin deficiency
f.
229.What is your initial management for this patient?
a. Oral rehydration solution 100-200mL every episode of diarrhea
b. Isotonic IV solution 30 ml/kg/0,5 hours, then 70 ml/kg/2,5 hours
c. Isotonic IV solution 100 ml/kg/3 hours
d. Oral rehydration solution 75 ml/kg/3hrs
e. Oral Rehydration ad libitum (as much as patient desired )

230.A 1-month-old baby presents with a diarrhea since 2 days before admission. He is having watery
diarrhea for 4-6 times per day with no blood no mucous; acid smell, and are not bulky or oily.
Her mother complained that her baby has rash around his buttock. What substance that is not
properly absorbed in the case above?
a. fats
b. proteins
c. carbohydrate
d. vitamins
e. minerals

231.A 60-year-old woman came to emergency room because of cramping abdominal pain, vomiting,
distended abdomen, and lack of defecation or flatus for the last 2 days. Abdominal examination
elicited abdominal distention and hyperactive of bowel sound.
What is the initial diagnostic test of the case above?

a. colonoscopy
b. abdominal CT-Scan
c. gastroscopy
d. ultrasonography
e. abdominal radiographs

232.A 8-year-old girl presents in the emergency department with a 24-hour history of increasingly
severe right lower quadrant abdominal pain. The pain began in the periumbilical region, which
was associated with nausea and vomiting. On physical examination, she has tenderness in the
right lower quadrant with rebound, and there is no mass. On rectal examination also reveals
right-sided tenderness. Low grade fever and mild leukocytosis are present.
What is the most likely diagnosis?

a. Acute diverticulitis
b. Acute appendicitis
c. Ureteral stone
d. Pyelonephritis
e. Intussusception

233.A 45-year-old man presents in the emergency room with a history of acute onset of severe
epigastric pain radiating toward the back. The pain began several hours after consuming fatty
meal. The patient has no significant medical history and denies any previous surgery. On
physical examination, the patient has marked epigastric tenderness with guarding and
hypoactive bowel sounds. The patient’s amylase level is 2500 units.
What is the most likely cause of this patient’s pain?

a. Gallstones
b. GERD
c. Duodenal ulcer
d. Nephrolithiasis
e. Peptic ulce

234.A 60-year-old male, come to the Siloam Hospital with a chief complain of bloody vomiting. The
blood was red and fresh. The patient also tells that his stool was dark. At physical examination,
there are palmar eritema, spider neavi, and ascites at the abdomen. The USG show a nodul on
the right lobule of the hepar.
Which of the following test that can help you diagnose?
a. CA 19-9
b. -feto protein
c. PSA
d. CA-153
e. Seromoeba

235.A 46-year-old Female, come to the clinic, with a chief complain of severe pain in the upper right
abdomen. the pain is not show up all the time, and it is spreading to the upper middle
abdomen. the patient also complains of nausea, vomiting and fever. On the physical
examination, her weight is 80kgs, height 160cm, on the abdomen, Murphy’s sign (+).
What is the most likely diagnosis?
a. Perforating of gastric ulcers
b. Acute Pancreatitis
c. Gastroduodentitis
d. Acute Cholesistitis
e. Acute Pyelonefritis

236.A mother bring her 3-month-old baby to the hospital with a chief complain of diarrhea 6 times a
day, since 2 days ago. She also tells that her baby was vomiting, and became irritable. She also
tells that she gave her baby powder cow milk (2 table spoon with 30cc of water) for 3 days,
before the baby get diarrhea. On the physical examination, the baby weight is 7 kg, Temp :
36.4OC, pulse: 110x/min, irritable, crying with small amount of tears, skin turgor slightly
decrease.
What is the type of diarrhea that the baby suffer?

a. Secretoric diarrhea
b. Traveller’s diarrhea
c. Exudative diarrhea
d. Osmotic diarrhea
e. Food poisoning diarrhea

237.A female tourist developed gastroenteritis while visiting small town in Indonesia and tried some
Indonesian traditional food. The onset of the disease is abrupt with abdominal cramps and
watery diarrhea. She had no fever or nausea or vomiting. The symptoms have resolved within
24 hour and no subsequent recurrences. They report the disease to district public health office.
The investigation found that one of the food products eaten by this tourist was contaminated by
suspected pathogens.What is the suspected pathogen may cause the disease above ?

a. Salmonella typhi
b. Shigella dysenteriae
c. Cryptosporidium parvum
d. Giardia lamblia
e. Enterotoxigenic E.coli
238. Approximately 4 hour after eating a meal in restaurant, 2 members of a college student group
develop a sudden onset of nausea, vomiting, dizziness and severe abdominal cramps. Nobody
got febrile and only 1 had diarrhea. Within a day, the symptoms has disappeared and no
recurrences.
239. Which of the following is the most important specimen to be collected to confirm diagnosis ?

a. Blood
b. Urine
c. Food
d. Fecal
e. Vomiting material

240. A 36 years old woman came to private family doctor complaining fever for 7 days, nausea,
vomiting, anorexia and diarrhea. Physical examination :
Temperature 39 °C, blood pressure 100/70 mmHg, pulse rate 80 per minute, respiratory rate 20
per minute. She was moderately ill-appearing, dry and white mucous membrane and tremor
tongue, Hiperactive bowel abdominal sound and mild diffuse tenderness
Laboratory findings : WBC count of 4500/mm3 , diff count 0/2/13/42/41/2
What organism may cause these symptomps above?

a. Shigella dysenteriae
b. Enteropathogenic E.coli
c. Salmonella enteritidis
d. Vibrio cholerae
e. Campylobacter

241.Mrs.X, 42 years old, came to emergency department with a mild fever, followed by malaise,
fatigue, nausea and vomiting for 5 days. Acholic stool and darkening urine are also appeared.
Sclera and the skin looked yellow. The liver is palpable 2 cm below the costal margin, tender and
the spleen is enlarged. The patient was unconscious. No prior history of the disease.
What is the mechanism of the darkening urine in the case above?

a. The unconjugated bilirubin filtered by the glomerulus


b. The red blood cells filtered by the glomerulus and appears in the urine
c. It is appears due to inflammation reaction in the renal
d. The unconjugated bilirubin is water soluble so it can appears in the urine
e. The conjugated bilirubin increase in plasma,readily filtered by the glomerulus

242. A 60 years old man, came to hospital with unconscious condition. From his relatives, we knew
that epigastric pain, nausea and heartburn were rare . Some medications ( Antacid etc ) had
been taken but there was no improvement. He has taken aspirin for his heart disease since 5
years ago. There was no hematemesis and no abnormality on the physical findings. Endoscopic
results showed some erosions and bleeding particularly in the anthrum.

a. What is the most recommended treatment to prevent this disease?


a. Mucoprotector
b. H-2 receptor blocker
c. COX-2 specific inhibitors
d. Proton Pump Inhibitor
e. Prostaglandin analogue

b. What is the laboratory finding might show?


a. Increased platelet
b. Decreased hemoglobine
c. Decreased white blood count
d. Increased AST
e. Prolonged APTT
c. What substance synthesis is impaired in the case above?
a. Kinin
b. Pepsin
c. Glycoprotein
d. Gastrin
e. Prostaglandin

243.A 25 year old man with typhoid fever and acute abdominal pain. The surgeon is examining the
small intestine during a surgical procedure. As a clerk in the surgery department, you have to
know the difference between jejunum and ileum
What is the most alteration between jejunum and ileum?

a. Diameter jejunum smaller than ileum


b. The wall of jejunum thicker than ileum
c. Vascularity of jejunum less than ileum
d. Color of jejunum pale that ileum
e. Plica circularis of jejunum shorter than ileum

244.A 69 year old man with abdominal mass underwent laparatomy. You are examining the large
intestine during the surgical procedure. To avoid cutting the vessel, you must know the
topography of the intestine.
What structure is supplied by the inferior mesenteric?
a. Colon descenden
b. Caudate ascenden
c. Colon transversum
d. Jejunum
e. Ileum
245. A 45- year old man was admitted to the emergency department complaining of severe pain in
the right lower quadrant of the anterior abdominal wall. He had repeatedly vomited, and his
temperature and pulse rate were elevated. On examination, the muscles of the lower part of
the anterior abdominal wall in the right lower quadrant showed tenderness at the MC Burney
point. You are at surgery for removal, but the organ is not visible. What position of this organ is
likely to be?
a. Paracecal
b. Paracolic
c. Retrocecal

d. Iliocecal
e. Retrocolic
246. A 42-year old male has a 6 months history of epigastric pain and is constant in nature especially
after meals. His passed tarry stool (black stool) over the previous 2 moths. Laboratory of stool’s
test showed occult blood.
What organ is the most likely to be affected?
a. jejunum
b. gaster
c. ileum
d. appendix
e. gallbladder
247. A 55 year old man with a long history of duodenal ulcer was seen in emergency room after
vomiting blood and exhibiting severe hypovolemic shock. Assuming the ulcer had perforated the
posterior wall of the first part of duodenum, which artery is most likely to have been eroded?
a. Splenic artery
b. Right gastric artery
c. Gastroduodenal artery
d. Right gastroepiploica artery
e. Proper Hepatic artery

248. A 17 year old healthy boy is having breakfast. He masticate the food slowly, to give attention to
his food in his mouth. He feels that the saliva is still coming out from the saliva’s glands, and
make mastication is going more gentle, the taste of the food more tasty. Then he swallow the
bolus. He can manage the action of swallowing.

a. What is the first reflex in swallowing food?


a. glottic closure
b. inhibition of respiration
c. relaxation of lower esophageal sphincter

d. contraction of esophageal muscles


e. contraction of pharyngeal muscles

What reflex allow the food to enter the stomach?


b. glottic closure
c. inhibition of respiration
d. relaxation of lower esophageal sphincter
e. contraction of esophageal muscles
f. contraction of pharyngeal muscles
249. A 15 year old boy is having a cup of coffee for the first time. After 30 minutes, he feel nausea
and then vomiting. During vomiting, he feels the muscles of his abdominal wall contract
forcefully and then the content of his stomach is push back to his mouth and to the outside. He
thinks that between stomach and the mouth during normal conditions there must be something
that prevent the content of the stomach to flow back to the mouth.
Which organ plays an important role in the prevention of gastric reflux?
a. glottis
b. diaphragm
c. sphincter pylorus
d. lower esophageal sphincter
e. peristaltic ring contraction of esophageal muscles

Вам также может понравиться